Exam 4

अब Quizwiz के साथ अपने होमवर्क और परीक्षाओं को एस करें!

A backcountry skier has been airlifted to the ED after becoming lost and developing hypothermia and frostbite. How should the nurse best manage the patient's frostbite? A) Immerse affected extremities in water slightly above normal body temperature. B) Immerse the patient's frostbitten extremities in the warmest water the patient can tolerate. C) Gently massage the patient's frozen extremities in between water baths. D) Perform passive range-of-motion exercises of the affected extremities to promote circulation.

A

A patient has been brought to the ED with multiple trauma after a motor vehicle accident. After immediate threats to life have been addressed, the nurse and trauma team should take what action? A) Perform a rapid physical assessment. B) Initiate health education. C) Perform diagnostic imaging. D) Establish the circumstances of the accident.

A

A patient is admitted to the ED with suspected alcohol intoxication. The ED nurse is aware of the need to assess for conditions that can mimic acute alcohol intoxication. In light of this need, the nurse should perform what action? A) Check the patient's blood glucose level. B) Assess for a documented history of major depression. C) Determine whether the patient has ingested a corrosive substance. D) Arrange for assessment of serum potassium levels.

A

Which patient should the nurse prioritize as needing emergent treatment, assuming no other injuries are present except the ones outlined below? A) A patient with a blunt chest trauma with some difficulty breathing B) A patient with a sore neck who was immobilized in the field on a backboard with a cervical collar C) A patient with a possible fractured tibia with adequate pedal pulses D) A patient with an acute onset of confusion

A

A 13-year-old is being admitted to the ED after falling from a roof and sustaining blunt abdominal injuries. To assess for internal injury in the patient's peritoneum, the nurse should anticipate what diagnostic test? A) Radiograph B) Computed tomography (CT) scan C) Complete blood count (CBC) D) Barium swallow

B

A 6-year-old is admitted to the ED after being rescued from a pond after falling through the ice while ice skating. What action should the nurse perform while rewarming the patient? A) Assessing the patient's oral temperature frequently B) Ensuring continuous ECG monitoring C) Massaging the patient's skin surfaces to promote circulation D) Administering bronchodilators by nebulizer

B

A male patient with multiple injuries is brought to the ED by ambulance. He has had his airway stabilized and is breathing on his own. The ED nurse does not see any active bleeding, but should suspect internal hemorrhage based on what finding? A) Absence of bruising at contusion sites B) Rapid pulse and decreased capillary refill C) Increased BP with narrowed pulse pressure D) Sudden diaphoresis

B

A 23-year-old woman is brought to the ED complaining of stomach cramps, nausea, vomiting, and diarrhea. The care team suspects food poisoning. What is the key to treatment in food poisoning? A) Administering IV antibiotics B) Assessing immunization status C) Determining the source and type of food poisoning D) Determining if anyone else in the family is ill

C

A patient with a fractured femur presenting to the ED exhibits cool, moist skin, increased heart rate, and falling BP. The care team should consider the possibility of what complication of the patient's injuries? A) Myocardial infarction B) Hypoglycemia C) Hemorrhage D) Peritonitis

C

A patient with multiple trauma is brought to the ED by ambulance after a fall while rock climbing. What is a responsibility of the ED nurse in this patient's care? A) Intubating the patient B) Notifying family members C) Ensuring IV access D) Delivering specimens to the laboratory

C

Define DKA

Caused by profound insulin deficiency that is characterized by: 1) hyperglycemia 2) ketosis 3) acidosis 4) dehydration and is likely to occur in type 1 diabetes.

Emergency Severity Index (ESI)

ESI-1: assess for threats to life (imminent danger of dying?) ESI-2: high-risk who should NOT wait ESI-3, ESI-4, ESI-5: # of anticipated resources needed

Define HHS

Life threatening syndrome (medical emergency with high mortality rate) that occurs in diabetics who are able to make enough insulin to prevent DKA, but not enough to prevent hyperglycemia, osmotic diuresis, and extracellular fluid depletion. Likely to occur in patients >60 with type 2. Causes: 1) newly diagnosed type 2 2) UTIs, pneumonia, sepsis, acute illness and (infection causes inflammation = release stress hormone causing BG to spike)

Define Diabetes Insipidus (DI)

Not enough ADH or low renal response to ADH causing increased urine output leading to F&E imbalance S/S: nocturia, generalized weakness, severe dehydration (tachy/hypotension) and hypovolemic shock Uncorrected hypernatremia: cause brain shrinkage and intracranial bleeding (irritability, mental dullness, coma) 1) polyuria and polydipsia > 200 mL/hr, 2-20L/day 2) urine specific gravity <1.005, urine osmolality <100 mOsm/kg 4) Hypernatremia >145 mEq/L, serum osmolality >295 mOms/kg

Following an addisonian crisis, a patient's adrenal function has been gradually regained. The nurse should ensure that the patient knows about the need for supplementary glucocorticoid therapy in which of the following circumstances? A) Episodes of high psychosocial stress B) Periods of dehydration C) Episodes of physical exertion D) Administration of a vaccine

a

A client with a history of coronary artery disease has developed diabetes insipidus as a result of cranial surgery. The client's medication therapy will include vasopressin (Pitressin). The nurse monitors this client most carefully for which sign/symptom that indicates an adverse effect of this medication? a. Depression b. Chest pain c. Joint stiffness d. Nagging cough

b

The patient with advanced cirrhosis ask why his abdomen is so swollen. The nurses responses based on the knowledge that a. a lack of clotting factors promotes the collection of blood in the abdominal cavity b. portal hypertension and hypoalbuminemia causes of fluid shift into the peritoneal space c. decrease peristalsis in the GI tract contributes to gas formation and distension of the bowel d. Bile salts in the blood irritate the peritoneal membranes, causing edema and pocketing of fluid

b

Which assessment finding would be increased in the patient with diabetes insipidus? a. Temperature b. urine output c. serum glucose d. BP

b

A 30-year-old female patient has been diagnosed with Cushing's syndrome. What psychosocial nursing diagnosis should the nurse most likely prioritize when planning the patient's care? A) Decisional conflict r/t treatment options B) Spiritual distress r/t changes in cognitive function C) Disturbed body image r/t changes in physical appearance D) Powerlessness r/t disease progression

c

Which disease is treated with corticosteroidal hormonal therapy? a. Thyrotoxicosis b. nephrotic syndrome c. adrenal insufficiency d. rheumatoid arthritis

c

Which goal of the treatment plan for a patient with DKA would be the initial focus? a. treatment for hypokalemia b. rapid reduction of elevated blood glucose c. rehydration through IV fluid replacement d. reduction of ketosis by encouraging oral nourishment

c

Which outcome is expected from the administration of midazolam? a. decreased vomiting b. analgesic c. conscious sedation d. relaxation of skeletal muscles for facilitation of endotracheal intubation

c

When caring for a patient with a life threatening injury, the nurse takes which action during the disability phase of the primary survey? a. Removes all clothing from the patient b. assesses the patient's airway and breathing c. evaluates the patient using the Glasgow coma scale GCS d. encourages the presence of the family during treatment

c. During the disability phase, a brief neurologic assessment is conducted using the GCS. Clothing is removed during the phase of "Exposure and Environmental Control." The patient's airway is initially assessed in the phase "Alertness and Airway." The presence of family is considered during the phase of "Facilitate Adjuncts and Family."

Would finding indicates mild hypothermia? a. body temperature < 86 degrees F b. body temperature of 86 - 93 degrees F c. body temperature of 93 - 95 degree F d. body temperature of 97 - 98 degree F

c. Mild hypothermia is 93 - 95 degree F. Moderate hypothermia is 86 - 93 degrees F. Severe hypothermia is < 86 degrees F

A patient with hypofunction of the adrenal cortex has been admitted to the medical unit. What would the nurse most likely find when assessing this patient? A) Increased body temperature B) Jaundice C) Copious urine output D) Decreased BP

d

What is the priority action for the nurse to take if the patient with type 2 diabetes reports blurred vision and irritability? a. Call the provider b. give insulin as ordered c. assess for other neurologic symptoms d. check the patient's blood glucose level

d

Which characteristic is seen in SIADH? a. Polyuria b. serum hyperosmolality c. dilutional hypernatremia d. fluid retention

d

Which condition would the nurse suspect when the patient's assessment findings include asterixis, hyperventilation, fetor hepaticus, and continuous lip smacking? a. Hepatitis b. liver cirrhosis c. hepatorenal syndrome d. hepatic encephalopathy

d

Which differentiates heat stroke from heat exhaustion? a. Fatigue b. Perspiration c. mental status d. core body temperature

d. In heat exhaustion, the core temperature of a patient is 99.6 to 105.8 degrees F and that of a patient with heat stroke is >105.8 degree F.

Pancreas Transplantation

Indication: type 1 DM with ESRD and kidney transplant (may be done same time) If successful, improves quality of life. -no need for exogenous insulin, BG monitoring and risk for hyper/hypoglycemia Lifelong immunosuppression Tx same as all transplant

Environmental Emergencies - Heat Stroke

MEDICAL EMERGENCY -Core temp. rises w/in 10-15 min, higher than 105.8 degree F -Hot, dry skin with no sweating, Circulatory Collapse -Altered mental status leading to confusion and coma -Hypotension, Tachycardia, Tachypnea, Weakness Interprofessional Care -stabilize ABCs -rapidly lower core temp. -monitor for dysrhythmias, continuous ECG -give 100% O2, BVM or intubate with mechanical vent., continuous pulse ox. Treatment -place in cool environment -remove clothes, cover w/sheets, spray with lukewarm water and place in front of large fan -immerse in cool water bath, apply ice to groin/axilla -refractory cases: peritoneal/rectal lavage with iced fluids -control shivering (with drugs) -monitor for rhabdomyolysis (check urine color, amount, pH and myoglobin) Education -Avoid future problems by properly hydrating on hot weather and exercises

Corticosteroid Therapy & Teaching

Side Effects: 1) increased glucose, delayed healing, risk of infection (suppressed immune response) 2) PUD, muscle wasting, protein depletion, mood face, truncal obesity, mood/behavior changes 3) osteoporosis 2) DO NOT STOP ABRUPTLY. - risk of acute adrenal crisis 3) Don't skip or double dose, taken with food in the morning

A patient admitted to the ED with severe diarrhea and vomiting is subsequently diagnosed with food poisoning. The nurse caring for this patient assesses for signs and symptoms of fluid and electrolyte imbalances. For what signs and symptoms would this nurse assess? Select all that apply. A) Dysrhythmias B) Hypothermia C) Hypotension D) Hyperglycemia E) Delirium

A, C, E

An ED nurse is triaging patients according to the Emergency Severity Index (ESI). When assigning patients to a triage level, the nurse will consider the patients' acuity as well as what other variable? A) The likelihood of a repeat visit to the ED in the next 7 days B) The resources that the patient is likely to require C) The patient's or insurer's ability to pay for care D) Whether the patient is known to ED staff from previous visits

B

Which dietary substance would the nurse recommend limiting for a patient with acute Cholecystitis? a. Fat b. protein c. calcium d. carbs

a

Define Triage

1) A French word meaning "to sort," refers to process of rapidly determining patient's acuity (critical assessment skill). Must treat patients who have a threat to life before other patients 2) Triage System -critical patients are treated first -Emergency Severity Index (ESI): 5-level system with triage algorithm to assign ESI levels -Illness severity and resource use to determine who should be treated first

Primary Survey - ABC

1) Airway/Alertness -airway obstruction caused by saliva, bloody secretion, vomit, laryngeal/facial trauma, fractures, dentures. S/S: dyspnea, cant speak, gasping (agonal breaths) -Risk for airway compromise: seizures, drowning, anaphylaxis, foreign body obstruction, cardiopulmonary arrest -Interventions: 1. jaw-thrust maneuver with suspected spinal cord injury (stabilize cervical spine and avoid hyperextension of neck). 2. Suction foreign body. 3. Naso/oropharyngeal airway for unconscious patients (also when breathing spontaneously when unconscious). 4. Ventilate with 100% O2 and BVM prior. (Endotrach intubation who cant be ventilated with oropharyngeal airway) 5. Emergency: cricothyroidotomy or tracheotomy 2) Breathing -Assess ventilation and breathing changes -assess for dyspnea, paradoxical/asymmetric chest wall movement, decreased breath sounds, chest wound, cyanonis, tachy, low BP. -Intervention: Tx underlying cause 1. BVM ventilation with 100% O2 2. Needle Decompression 3. Intubation 3) Circulation -Risk for hemorrhage: assess carotid or femoral pulses, skin, signs of shock -Insert 2 large-bore IV catheters: use upper extremities -Initiate aggressive fluid resus with NS or LR, blood products/PRBCs (use O negative for emergency transfusion.

Abnormalities of Liver and Gallbladder

1) Bile Duct obstruction -bile reabsorbed into blood and carried throughout entire body and excreted through urine causing deep orange, foamy, dark amber urine -steatorrhea (fatty stools because there is no bile to break down fat) 2) Cirrhosis: degeneration and destruction of liver cells causing scarring -Portal hypertension, hepatic encephalopathy, ascites -spider angiomas (vascular lesions) -varices in esophageal, gastric and rectal 3) Jaundice -bilirubin removed through body causing yellowing of sclera of the eyes, fingernails, palms of hands and oral mucosa 4) Cholecystitis -inflammation of gall bladder -avoid fatty foods -treat pain and infection -surgery may be needed

Clinical Manifestations

1) Cirrhosis -Fatigue with normal liver function (early symptoms) -Jaundice, peripheral edema, ascites, skin lesions, hematologic and endocrine problems, peripheral neuropathies (seen in alcoholics with diet deficiencies of thiamine, folic acid and cobalamin) 2) Skin lesions -caused by increase circulating estrogen and livers inability to metabolize steroid hormones -spider angiomas: bright red center point and spiderlike branches -palmar erythema: red area that blanches with pressure on palm of hands 3)Hematologic Disorders -thrombocytopenia, leukopenia, anemia (splenomegaly from backup of blood from portal vein) -coagulation disorders (liver cant make prothrombin and other blood clotting factors) causing epistaxis, purpura, petechiae, bruising, gingival bleeding, heavy menses 4)Endocrine Disorders -Men: gynecomastia, loss of axillary and pubic hair, testicular atrophy, impotence and loss of libido -Women: amenorrhea or vaginal bleeding -Hyperaldosteronism: sodium and water retention and potassium loss 5)Peripheral Neuropathy: sensory and motor symptoms

Primary Survey - DEFG

1) Disability -Assess neurologic status -Glasgow Coma Scale -Pupils (size, shape, equality and reactivity) 2) Exposure and Environmental Control -remove clothes to perform physical assessment -Dont remove impaled objects -Prevent hypothermia with warm blankets, warm IV fluids and overhead warmers -Maintain privacy -VS, O2 sat, BP in both arms 3) Facilitate adjuncts and Family -family presence during resus and invasive procedures 1. benefit for patient, caregivers and staff 2. provide comfort, advocate and remind staff of person behind trauma 3. serve as patient and staff helpers 4. assign someone to explain care and answer questions 4) Get resuscitation adjuncts 5) Uncontrolled external hemorrhage: <C> stands for "catastrophic hemorrhage" -ABC reprioritized to <C>ABC (catastrophic hemorrhage before airway, breathing, circulation) 1. Check pulse 1st while looking for signs of breathing at the same time. 2. Apply direct pressure until bleeding is controlled. Then apply pressure dressing

Secondary Survey

1) Head, neck, face -assess for lacerations, deformities, tenderness, bleeding, hematomas, drainage, ecchymosis, edema, foreign bodies -assess disconjugate gaze (neurologic damage) -Battle's sign or Raccoon eyes (skull fracture) -assess ears for blood and CSF (dont block clear draining from ear/nose) 2) Chest -Inspect: RR, depth, and effort (chest wall movement and use of accessory muscles) -Palpate: subcutaneous emphysema -Auscultate: breath sounds (possible pneumothorax, open pneumothorax, hemothorax, rib fracture, pulmonary contusion, blunt cardiac injury) -Obtain CXR and ECG to check external injuries 3) Abdomen and Flanks -Check frequently for subtle changes: blunt/penetrating trauma (immobilize until spinal injury R/O), external signs of injury, listen to bowel sounds, palpate masses, guarding and femoral pulses -Abdominal wounds: from gunshot wound and organ injuries depend on severity with mortality 5% from hemorrhage or infection 4) Extremities -observe external injury (immobilize/splint and elevate, apply ice packs), check pulses -compartment syndrome: compromises muscles, nerves and arteries 5) inspect posterior surface -logroll patient (protect cervical spine) -need 3-4 people -inspect and palpate for deformities, bleeding, lacerations, bruising, palpate entire spine

Death in ED

1) Identify/manage own reaction/feelings 2) Help family/friends begin grieving -comfort, privacy, offer chaplain visit (if appropriate), collect personal belongings, mortuary arrangements 3) May need to contact medical examiner/coroners for autopsies that family request of death within 24 hours of ED admission or when suspected trauma/violence/unusual death occurred. 4) Consider non-heart-beating donation: corneas, heart valves, skin, bone, kidneys. -Organ Procurement Organizations (OPOs) assist with screening, counseling donor families, harvest organs

Functions of Liver, Gallbladder, Pancreas, Duodenum

1) Liver (Hepatic Ducts) -Filters blood coming from digestive tract by detoxifying chemicals and metabolizing drugs -Metabolize steroid hormones: estrogen, testosterone, aldosterone -Makes proteins, prothrombin (for clotting) -storage of nutrients, glycogen and plasma protein synthesis (albumin) -secretes/excrete bile and bilirubin -synthesize cholesterol -Abnormalities: liver failure, cirrhosis, ascites, jaundice 2) Gall Bladder (Cystic Duct) -stores bile, concentrates it (causes gall stones) -Abnormalities: cholecystitis, cholelithiasis 3) Pancreas -secretes enzymes (lipase), bicarbs, exocrine and endocrine functions -Abnormalities: exocrine and endocrine 4) Duodenum -Connected to liver and pancreas via ducts -Abnormalities: ulcer

Care of Emergency Patient

1) Primary Survey: ABCDEFG 2) Secondary Survey History and head-to-toe assessment are brief, systemic way to look for ALL injuries

Kidney Transplant Complication

1) Rejection -Hyperacute: minutes to hours (anaphylactic reaction - mainly caused by patient not immunosuppressed enough) -acute: months and S/S: pain at site, flu-like symptoms, fever, wt. changes, swelling, low urine output, changes in HR) -chronic: months or years, irreversible and need to go back on transplant list 2) Infection -R/T suppression of immune response, immunosuppressant drugs, effects of ESRD -Most commonly seen during 1st month: pneumonia, wound infection, IV line/drain, UTIs -Fungals: candida, Cryptococcus , Aspergillus, Pneumocystis jiroveci -Virals: CMV, Epstein-Bar, HSV, Varicella-zoster 3) CVD -increased incidence of atherosclerotic vascular disease. Immunosuppressants cause HTN and hyperlipidemia (adhere to antihypertensive meds) 4) Cancers -due weak immune systems from immunosuppressed drugs -skin cancer most commonly seen -Screening and wear protective clothes and sunscreen. 5) Corticosteroid-related -Necrosis: hips, knees, joints -PUD, infection -Diabetes, cataract -Dyslipidemia, Cancer

Nursing Intervention

1) Rest 2) Administer B-Complex vitamin (for peripheral neuropathy in alcoholic cirrhosis) 3) Avoid alcohol 4) Esophageal and gastric varices: Reduce risk of bleeding: minimize use/avoid aspirin, acetaminophen and NSAIDs, screen for presence with endoscopy, nonselective B-blockers (propanalol) -If bleeding occurs, stabilize and manage airway, start IV therapy and blood products (FFP, PRBCs, vitamin K) Vasopressin to constrict blood vessels to stop bleeding 5) Treat Ascites -sodium restriction -diuretics (with caution because patient already hypokalemic, spironolactone 1st choice) -fluid removal (possibly several times/day) -Albumin supplement -Tolvaptan (Samsca) to inhibit ADH

S/S of Cushing Syndrome

1) Too much glucocorticoids (cortisol): - hyperglycemia, wt. gain, delay wound healing, thin skin and easily bruised -muscle wasting: weakness, osteoporosis, back pain, hypercalciuria (urine calcium excretion) - Tx with foods high in vitamin D, protein and calcium, fall precaution - truncal obesity, moon face, acne, abdominal striae 2) Too much mineralocorticoid (aldosterone) - hypokalemia and hypertension: monitor BP, K levels and urine output 3) Too much adrenal and androgen - severe acne, male characteristics in women, feminization in men

Kidney Transplant

1) Very successful and best treatment for ESRD (reverses pathophysiology) 2) No dialysis needed and no dietary/lifestyle restriction 3) Immunosuppressive Therapy goals -suppress immune response to prevent rejection -sufficient immunity to prevent overwhelming infection

A patient is admitted to the ED after being involved in a motor vehicle accident. The patient has multiple injuries. After establishing an airway and adequate ventilation, the ED team should prioritize what aspect of care? A) Control the patient's hemorrhage. B) Assess for cognitive effects of the injury. C) Splint the patient's fractures. D) Assess the patient's neurologic status.

A

An 83-year-old patient is brought in by ambulance from a long-term care facility. The patient's symptoms are weakness, lethargy, incontinence, and a change in mental status. The nurse knows that emergencies in older adults may be more difficult to manage. Why would this be true? A) Older adults may have an altered response to treatment. B) Older adults are often reluctant to adhere to prescribed treatment. C) Older adults have difficulty giving a health history. D) Older adults often stigmatize their peers who use the ED.

A

The triage nurse is working in the ED. A homeless person is admitted during a blizzard with complaints of being unable to feel his feet and lower legs. Core temperature is noted at 33.2∞C (91.8∫F). The patient is intoxicated with alcohol at the time of admission and is visibly malnourished. What is the triage nurse's priority in the care of this patient? A) Addressing the patient's hypothermia B) Addressing the patient's frostbite in his lower extremities C) Addressing the patient's alcohol intoxication D) Addressing the patient's malnutrition

A

A patient is brought by friends to the ED after being involved in a motor vehicle accident. The patient sustained blunt trauma to the abdomen. What nursing action would be most appropriate for this patient? A) Ambulate the patient to expel flatus. B) Place the patient in a high Fowler's position. C) Immobilize the patient on a backboard. D) Place the patient in a left lateral position.

C

A patient is brought to the ED by ambulance after swallowing highly acidic toilet bowl cleaner 2 hours earlier. The patient is alert and oriented. What is the care team's most appropriate treatment? A) Administering syrup of ipecac B) Performing a gastric lavage C) Giving milk to drink D) Referring to psychiatry

C

A patient is experiencing respiratory insufficiency and cannot maintain spontaneous respirations. The nurse suspects that the physician will perform which of the following actions? A) Insert an oropharyngeal airway. B) Perform the jaw thrust maneuver. C) Perform endotracheal intubation. D) Perform a cricothyroidotomy.

C

While performing triage in the ED, the nurse determines which patient should be seen first? a. The patient with burns on the face and chest; BP 120 / 80 mmHg, HR 92, RR 24 b. the patient with a deformed leg indicating a fractured tibia; BP 110 / 60 mmHg, HR 86, RR 18 c. the patient with type 1 diabetes and ketoacidosis; BP 100 / 60 mmHg, HR 100, RR 32 d. a patient with a respiratory infection with a cough productive of greenish sputum; BP 128 / 86 mmHg, HR 88, RR 26

C

You are a floor nurse caring for a patient with alcohol withdrawal syndrome. What would be an appropriate nursing action to minimize the potential for hallucinations? A) Engage the patient in a process of health education. B) Administer opioid analgesics as ordered. C) Place the patient in a private, well-lit room. D) Provide television or a radio as therapeutic distraction

C

Organ Transplantation

Ch. 13

Conscious Sedation

Chapter 18 Now called moderate sedation/analgesia -sedative, anxiolytic and/or analgesic drugs -Benzos: diazepam, lorazepam -S/E: respiratory depression, hypotension, tachycardia -ANTIDOTE: flumazenil -Not expected to induce level of sedation that would impair ability to protect airway so patient is responsive and breathes without assistance -used for minor therapeutic procedures -NEVER leave unattended, and not use of inhaled agents

Liver, Gallbladder, Pancreas, Duodenum

Chapter 43

Emergency Care Situations

Chapter 68

Environmental Emergencies - Poisoning

Chemicals that harm body -accidental, occupational, recreational, international -natural or manufactured toxins ingested, inhaled, splashed in eyes or absorbed through skin -come from toxic plants or contaminated food -Healthcare workers at risk for exposure to hazardous materials Treatment -Poison Control Center Guidance: bind agents to eliminate with: activated charcoal, cathartics, whole-bowel irrigation, hemodialysis, chelating agents, antidotes Types of Poisons 1) Acids and alkali -dilute with water/milk -gastric lavage performed within 1 hr of ingestion -NEVER induce vomiting (contraindicated) or use syrup ipecac 2) Bleaches -wash exposed skin and eyes -dilute with water/milk -gastric lavage -PREVENTION of vomiting and aspiration 3) NSAIDs -activated charcoal, gastric lavage, supportive care 4) Acute alcohol toxicity -CNS suppression, hypokalemia, hypomagnesemia, hypoglycemia (mimic alcohol intoxication and should be assessed - give glucose-containing IV solutions) -No antidote, supportive care measures and maintain ABCs until alcohol metabolizes -frequently monitor VS and LOC

Environmental Emergencies - Hypothermia

Core temperature <95 degree F Factors: a) exposed to freezing temp., cold winds, wet terrain, physical exhaustion, inadequate clothing, inexperience. b) Older adults decreased: body fat, energy reserves, metabolic rate, shivering response. Chronic medical conditions, drugs alter body defenses. Findings -Decreased core temp. -cold myocardium causes dysrhythmias (VFIB, Brady, PEA, Asystole) -Decreased renal blood flow = dehydration -Decreased blood flow = hypoxia, anaerobic metabolism, lactic acid buildup, metabolic acidosis -Increased HCT (cold blood is thick = risk for stroke, MI, PE and renal failure) Severe hypothermia (life-threatening) findings: - core temp. <86 degree F -undetectable VS (person appears dead) -absent reflexes, fixed/dilated pupils -ECG: bradycardia, VFIB, PEA -Every effort to warm patient to at least 86 degree F before pronouncing dead (cause of death is refractory VFib) Treatment -remove from cold environment -maintain ABCs and provide O2 -IV access for fluid resus -correct dehydration and acidosis -12 lead ECG and treat dysrhythmias

A patient is brought to the ER in an unconscious state. The physician notes that the patient is in need of emergency surgery. No family members are present, and the patient does not have identification. What action by the nurse is most important regarding consent for treatment? A) Ask the social worker to come and sign the consent. B) Contact the police to obtain the patient's identity. C) Obtain a court order to treat the patient. D) Clearly document LOC and health status on the patient's chart.

D

An obtunded patient is admitted to the ED after ingesting bleach. The nurse should prepare to assist with what intervention? A) Prompt administration of an antidote B) Gastric lavage C) Administration of activated charcoal D) Helping the patient drink large amounts of water

D

Complications

For cirrhosis -Portal Hypertension: splenomegaly, large collateral veins, ascites -Varices (most life-threatening complication) a) esophageal varices: enlarged veins at lower end of esophagus b) gastric varices on upper part of stomach (very fragile and bleed easily. Avoid alcohol, aspirin and NSAIDs) 1)Peripheral Edema 2) Ascites caused by: portal hypertension (fluid leak into peritoneal cavity), hypoalbuminemia (fluid shift into interstitial space from lack of synthesized albumin) and hyperaldosteronism (retention) 3)Hepatic encephalopathy caused by neurotoxic effects of ammonia -Manifestation: -changes in neurologic and mental responsiveness, impaired consciousness, inappropriate behavior, sleep disturbances, trouble concentrating, coma. (Treat with lactulose) -Asterixis: flapping tremors in arms/hands, not unable to hold arms/hands stretched out -Fetor hepaticus: musty, sweet odor of patient's breath

S/S of SIADH

S/S 1) ↓ urine out and ↑ body wt. 2) initial: thirsty, dyspnea, fatigue 3) Mild (>120 mEq/L): muscle cramps, headache, irritable 4) Severe (<120 mEq/L): vomiting, abdominal cramps, muscle twitching 5) EMERGENCY: cerebral edema causing lethargy, confusion, seizures and coma

Environmental Emergencies - Frostbite

Tissue freezing leads to ice crystals in tissues and cells -vasoconstriction from cold stress leads to decreased blood flow Superficial Frostbite -skin and subcutaneous tissues (ears, nose, fingers, toes) -waxy pale yellow to blue mottled skin -crunchy and frozen Treatment -immense affected area in circulating water 98.6-104 degree F (slightly above body temp) for 30-40 min -warm soaks for facial areas -debride and apply sterile dressing -analgesia (rewarming extremely painful and stings as it thaws and blisters form w/in hours) and tetanus prophylaxis -AVOID heavy blankets and clothing NEVER squeeze, massage or scrub injured tissue -Remove clothing and jewelry because swelling happens with thawing.

Define Cushing Syndrome

Too much corticosteroid exposure Caused by: 1) mainly exogenous (prednisone) 2) 85% endogenous cases caused by ACTH-secreting pituitary adenoma (Cushing's Disease)

Nursing Intervention for Cushing Syndrome

Treat the cause!! 1) prolong use of prednisone -gradually D/C -alternate day dosing 2) Pituitary Adenoma -surgical removal or radiation 3) adrenal tumors or hyperplasia -adrenalectomy (can cause release of stored catecholamines: monitor BP, change in HR and treat with nitroprusside and alpha-adrenergic blocking agents)

A patient has a serum sodium level of 152 mEq/L. The normal hormonal response to the situation is a. release of ADH b. release of ACTH c. secretion of aldosterone d. secretion of corticotropin-releasing hormone

a

An elderly woman reports weakness, abdominal pain, and vaginal bleeding. The patient's diagnostic test reports describe anemia and an enlarged spleen. Which condition is consistent with these findings? a. Liver cirrhosis b. pancreatic cancer c. acute liver failure d. acute pancreatitis

a

An older man arrives in triage disoriented and dyspneic. Skin is hot and dry. his wife states that he was fine earlier today. The nurses next priority would be to a. assess his vital signs b. obtain a brief medical history from his wife c. start supplemental O2 and have the provider see him d. determine the kind of insurance he has before treating him

a

Six days after kidney transplant from a deceased donor, a patient develops a temperature of 101.2 degrees Fahrenheit, tenderness at the transplant site, and oliguria. The nurse recognizes that these findings indicate a. acute rejection, which is not uncommon and is usually reversible b. hyperacute rejection, which will necessitate removal of the transplant kidney c. all infection of the kidney, which can be treated with IV antibiotics d. the onset of chronic rejection of the kidney with eventual failure of the kidney

a

The nurse is providing care for a patient with Cushing's syndrome has identified the nursing diagnosis of risk for injury r/t weakness. How should the nurse best reduce this risk? A) Establish falls prevention B) Encourage bed rest whenever possible C) Encourage the use of assistive devices D) Provide constant supervision

a

To control the side effect of corticosteroid therapy, the nurse teaches the patient who is taking corticosteroids to: a. increase calcium intake to 1500 milligram/day b. perform glucose monitoring for hypoglycemia c. obtain immunizations due to high risk for infection d. avoid abrupt position changes because of orthostatic hypotension

a

You are developing a care plan for a patient with Cushing's syndrome. What nursing diagnosis would have the highest priority in this care plan? A) Risk for injury r/t weakness B) Ineffective breathing pattern r/t muscle weakness C) Risk for loneliness r/t disturbed body image D) Autonomic dysreflexia r/t neurologic changes

a

Which assessment findings are associated with heat exhaustion? SATA a. Dilated pupils b. Tachycardia c. Hypertension d. flush skin e. muscle contractions

a, b Prolonged exposure to heat over hours or days leads to heat exhaustion. This is a clinical syndrome characterized by fatigue, nausea, vomiting, extreme thirst, and feelings of anxiety. Tachycardia and dilated pupils are present.

When preparing to cool a patient who is to begin therapeutic hypothermia, which intervention will the nurse plan to do (select all that apply)? a. Assist with endotracheal intubation. b. Insert an indwelling urinary catheter. c. Begin continuous cardiac monitoring. d. Obtain an order to restrain the patient. e. Prepare to give sympathomimetic drugs.

a, b, c Cooling can produce dysrhythmias, so the patients heart rhythm should be continuously monitored and dysrhythmias treated if necessary. Bladder catheterization and endotracheal intubation are needed during cooling. Sympathomimetic drugs tend to stimulate the heart and increase the risk for fatal dysrhythmias such as ventricular fibrillation. Patients receiving therapeutic hypothermia are comatose or do not follow commands so restraints are not indicated.

Which actions will the nurse anticipate taking for a client who has been working outside on a sunny summer day and is now minimally responsive and has hypotension, body temperature of 106 degrees F, and dry skin? SATA a. Administer 100% 02 b. place in a cool bath c. administer cool IV fluids d. cover the patient with light blankets e. administer acetaminophen

a, b, c The patient data indicates heat stroke. Treatment focuses first on stabilizing the patient's ABCs and rapidly reducing the core temperature. Administration of 100% O2 compensates for the patients hypermetabolic state. Cooling of the body with IV infusions will help to rapidly lower the temperature. Immersion in a cool water bath will increase conductive heat loss. The patient will be uncovered to increase heat loss. Acetaminophen will not be effective because the increase in temperature is not related to infection or problems with the hypothalamic set point for temperature.

A nurse is teaching a client who is scheduled for a kidney transplant about organ rejection. Which of the following statements should the nurse include? SATA a. "Expect immediate removal of the donor kidney for hyperacute rejection." b. "You might need to begin dialysis to monitor your kidney function for a hyperacute rejection." c. "A fever is a manifestation of an acute rejection." d. "Fluid retention is a manifestation of an acute rejection." e. "Your provider will increase your immunosuppressive medication for a chronic rejection."

a, c, d

A patient with Cushing's syndrome has been hospitalized after a fall. The dietitian consulted works with the patient to improve the patient's nutritional intake. What foods should a patient with Cushing's syndrome eat to optimize health? Select all that apply. A) Foods high in vitamin D B) Foods high in calories C) Foods high in proteins D) Foods high in calcium E) Foods high in sodium

a, c, d

Which assessment findings are associated with complications of cirrhosis of the liver? SATA a. pedal edema b. productive cough c. mental status changes d. black, tarry stools e. chest pain with diaphoresis

a, c, d, Complications of cirrhosis of the liver include peripheral edema, gastric varices, and hepatic encephalopathy. Peripheral edema presents itself as swelling/edema of the feet, or pedal edema.

Which guidance would the nurse provide to a patient diagnosed with cirrhosis whose abdomen is distended and has a visible fluid wave? SATA a. Avoid alcohol b. See a nephrologist c. restrict sodium in the diet d. take Lactulose as directed e. expect urinary frequency with spironolactone

a, c, d, e Management of cirrhosis focuses on slowing the progression of disease while preventing or treating complications. Abdominal distension and the visible fluid wave signs of ascites, a complication of cirrhosis and an indication of disease progression. Cessation of alcohol consumption will prevent further damage to the liver. Sodium restriction is a hallmark of ascites management and can help prevent the reaccumulation of fluid. Lactulose is used to treat hepatic encephalopathy and manage ammonia levels. Frequent loose stools results, and patients very often need to be encouraged to take the medication as directed. Spironolactone is an aldosterone antagonist and potassium-sparing diuretic that is frequently used to help manage ascites. The nurse will discuss the therapeutic effect with the patient, which is increased urination. Referral to nephrology is not likely to be necessary at this point as there is nothing to suggest impaired kidney function.

A patient with cirrhosis of the liver is admitted to the hospital. Which hematologic symptoms are likely in this patient? SATA a. Anemia b. Leukemia c. Leukopenia d. polycythemia Vera e. thrombocytopenia

a, c, e

Which topics would the nurse discuss with the patient and family, prior to the scheduled kidney transplant, when the live donor cross match results are positive? SATA a. The kidney transplantation cannot be done b. proceeding with the transplantation is safe c. a renal scan is needed to confirm the ability to continue with transplantation d. if transplanted, the organ would undergo a hyperacute rejection e. chronic rejection of the kidney is anticipated, but transplantation is safe for now

a, d A positive cross match indicates that the recipient has cytotoxic antibodies to the donor and is an absolute contraindication to transplantation. If transplanted, the organ will undergo hyperacute rejection. It is not safe to proceed with transplantation if the cost match is positive. A Reno scan will not help in this procedure.

Which injured children will be given a yellow tag by first responders at a school bus crash? SATA a. A child with a broken leg b. a child with minor scrapes c. a child in respiratory distress d. a child with a bruised and swollen arm e. the child complaining of abdominal pains f. a child who is drowsy and has a head wound

a, d The children with urgent non-life-threatening injuries are given yellow tags; these include the children with the broken leg and a child with a bruised and swollen arm. The children in respiratory distress, with abdominal pains, and with a head wound and drowsiness need red tags and will be transported to the hospital first. The child with minor scrapes will be given a green tag, indicating minor injuries, and will be transported last.

Which information would the nurse provide the patient and family waiting for an organ transplant? SATA a. Two organs can be transplanted together b. living donors can donate only a part of the organ c. patients are matched to the available donors according to their age d. Segments of organs can be transplanted instead of the complete organ e. an imminent death of a donor or a patient, the legal next of kin can consent for organ donation

a, d, e

Which interventions will the nurse expect in the treatment plan for a patient who's admitted to the hospital with ascites? SATA a. Anticipate paracentesis b. provide a high sodium diet c. encourage high fluid intake d. monitor fluid and electrolytes e. administer an albumin infusion

a, d, e

While in the emergency department, a patient dies unexpectedly as a result of complications from an injury. Which action does the nurse take when initially approaching the patient's family? SATA a. Provide support b. inquire about organ donation c. gather the patient's belongings d. discuss Mortuary arrangements e. provide family members with a place of privacy

a, e It is important to provide the family with support and privacy. Because the death was unexpected, it is likely that the family members will be in shock or disbelief. They will need support from the health care team, who also need to respect their privacy. Inquiring about organ donation should occur soon after the patient's death but not during the nurse's initial encounter with the family. Discussing mortuary arrangements and gathering the patient's belongings can occur later.

The nurse reviews the medical record of a patient and identifies which factor that contributes to cold related injuries? a. Age b. Obesity c. Dehydration d. physical exertion

a. Age is a contributing factor to cold related injuries. Older adults are more vulnerable to cold related injuries because of low body fat, low energy reserves, pre-existing chronic illness, and use of medicines that can alter their response mechanism in an event of adverse environmental conditions.

A patient in the ER asks the nurse whether he/she will need anesthesia for surgical reduction of displaced shoulder. Which response by the nurse is accurate? a. "You will most likely be moderately sedated for the procedure." b. "There is no need for anesthesia because the reduction does not involve an incision." c. "Due to your age, you will likely be given a general anesthetic before reduction." d. "The primary care provider will most likely use an epidural or spinal block to numb the area for the procedure."

a. Moderate to deep sedation is generally used for minor therapeutic procedures, such as fractures, in the ER.

After stabilization of her trauma victim's ABCs, which is the nurses next priority action? a. Assessing neurological status b. obtaining a complete history c. examining the extremities for fractures d. performing an abdominal assessment

a. Standard trauma care includes assessing and stabilizing the ABCs, followed by neurological assessment and care. This includes stabilizing the cervical spine. Obtaining a complete history, performing an abdominal assessment, and examining the extremities for fractures are done after assessing their ABC's and neurological status. The abdominal assessment examination of the extremities are part of the head to toe survey that is part of the secondary survey.

The nurse creates a plan of care for a patient with frostbite of the hands. The nurse identifies that which outcome corresponds with this condition? a. Brisk capillary refill b. adequate dietary intake c. balance fluid intake and output d. BP within normal limits

a. The major dysfunction with frostbite is impaired circulation. Therefore measures to promote and maintain adequate circulation are the highest priority.

A 23-year-old has been admitted with acute liver failure. Which assessment data are most important for the nurse to communicate to the health care provider? a. Asterixis and lethargy b. Jaundiced sclera and skin c. Elevated total bilirubin level d. Liver 3 cm below costal margin

a. The patients findings of asterixis and lethargy are consistent with grade 2 hepatic encephalopathy. Patients with acute liver failure can deteriorate rapidly from grade 1 or 2 to grade 3 or 4 hepatic encephalopathy and need early transfer to a transplant center. The other findings are typical of patients with hepatic failure and would be reported but would not indicate a need for an immediate change in the therapeutic plan.

Which type of anesthesia is a nurse able to administer without the presence of an anesthesia care provider (ACP)? a. moderate sedation b. general anesthesia c. regional anesthesia d. monitored anesthesia care

a. An RN who is educated in moderate sedation is permitted by institution protocols and state nurse acts can perform this. General anesthesia, regional anesthesia, and monitored anesthesia care require presence of an ACP.

Which event is defined as requiring a rapid and skilled medical response that existing response resources can manage easily? a. Emergency b. natural disaster c. acts of terrorism d. mass casualty incident

a. An emergency is defined as an event that requires a rapid and skilled medical response that existing resources can manage easily. A natural disaster is one type of mass casualty incident and usually requires more resources than the community has available and requires outside assistance. Terrorism involves overt actions, such as the use of nuclear, biologic, or chemical weapons, to cause harm and will involve federal agencies in the response. Mass casualty incidents can be natural or human-made but will require assistance from outside resources and agencies.

A patient with advanced cirrhosis ask why the skin is so yellow. Which information should the nurse use to provide a response? a. Jaundice results from the bodies inability to conjugate and excrete bilirubin b. a lack of clotting factors promotes the collection of blood under the skin surface c. decrease peristalsis in the GI tract contributes to an excess of bile salts d. decreased colloidal oncotic pressure from hypoalbuminemia causes the skin discoloration

a. Jaundice results from the functional derangement of liver cells and compression of bile ducts by connective tissue overgrowth. Jaundice occurs as a result of the decreased ability to conjugate and excrete bilirubin. Jaundice is not caused by a lack of clotting factors, and excess of bowel salts, or decrease colloidal oncotic pressure.

A 20-year-old patient arrives in the emergency department (ED) several hours after taking 25 to 30 acetaminophen (Tylenol) tablets. Which action will the nurse plan to take? a. Give N-acetylcysteine (Mucomyst). b. Discuss the use of chelation therapy. c. Start oxygen using a non-rebreather mask. d. Have the patient drink large amounts of water.

a. N-acetylcysteine is the recommended treatment to prevent liver damage after acetaminophen overdose. The other actions might be used for other types of poisoning, but they will not be appropriate for a patient with acetaminophen poisoning.

Which part of the assessment does the nurse address during the secondary survey of the patient in triage? a. Allergies b. Patency of the patient's airway c. neurologic status and level of consciousness d. presence or absence of breath sounds

a. Patient allergies are assessed during secondary survey. Airway, breathing, circulation, and a brief neurologic assessments are components of the primary survey that is performed to identify life threatening conditions.

After completing the primary and secondary assessments and providing prescribed treatment for a patient who has a traumatic injury, which action by the nurse has highest priority? a. Continue to monitor the patient for changes b. clean and restock the room for the next patient c. transfer the patient as soon as possible to the inpatient unit d. delegate appropriately to UAP

a. The highest priority is to continue to monitor the patient for changes and respond appropriately if changes in the assessment occur.

Which finding indicates to the nurse that lactulose (Cephulac) is effective for a 72-year-old man who has advanced cirrhosis? a. The patient is alert and oriented. b. The patient denies nausea or anorexia. c. The patients bilirubin level decreases. d. The patient has at least one stool daily.

a. The purpose of lactulose in the patient with cirrhosis is to lower ammonia levels and prevent encephalopathy. Although lactulose may be used to treat constipation, that is not the purpose for this patient. Lactulose will not decrease nausea and vomiting or lower bilirubin levels.

A patient is admitted to the hospital with acute rejection of a kidney transplant. Which intervention will the nurse prepare for this patient? a. Administration of immunosuppressant medications b. Insertion of an arteriovenous graft for hemodialysis c. Placement of the patient on the transplant waiting list d. A blood draw for human leukocyte antigen (HLA) matching

a. Acute rejection is treated with the administration of additional immunosuppressant drugs such as corticosteroids. Because acute rejection is potentially reversible, there is no indication that the patient will require another transplant or hemodialysis. There is no indication for repeat HLA testing.

The nurse teaches a patient about drug therapy after a kidney transplant. Which statement by the patient would indicate a need for further instructions? a. After a couple of years, it is likely that I will be able to stop taking the cyclosporine. b. If I develop an acute rejection episode, I will need to have other types of drugs given IV. c. I need to be monitored closely because I have a greater chance of developing malignant tumors. d. The drugs are given in combination because they inhibit different ways the kidney can be rejected.

a. Cyclosporine, a calcineurin inhibitor, will need to be continued for life. The other patient statements are accurate and indicate that no further teaching is necessary about those topics.

The nurse discusses frostbite with a group of nursing students and provides which description of the condition? a. It is tissue damage due to freezing b. it is a deficiency of oxygen in the tissues c. it is a core temperature between 93 to 95 degree F d. it occurs when heat produced by the body cannot compensate for heat loss to the environment

a. Frostbite is cold related injury caused by exposure to extreme weather. The body tissues freeze, which results in formation of ice crystals in the tissues and cells.

A 56-year-old female patient has an adrenocortical adenoma, causing hyperaldosteronism. The nurse providing care should a. monitor the blood pressure every 4 hours. b. elevate the patients legs to relieve edema. c. monitor blood glucose level every 4 hours. d. order the patient a potassium-restricted diet.

a. Hypertension caused by sodium retention is a common complication of hyperaldosteronism. Hyperaldosteronism does not cause an elevation in blood glucose. The patient will be hypokalemic and require potassium supplementation before surgery. Edema does not usually occur with hyperaldosteronism.

Which part of the body is most vulnerable to frostbite in a superficial injury? a. Skin b. bone c. Muscle d. tendon

a. In a superficial frostbite, the surface of the skin is affected. The appearance of the skin varies from waxy pale to blue to modelled in color. Dark skinned persons are at a higher risk, making them more prone to frostbite. And deep frostbite, the bones muscle and tendons are involved in the freezing process. The patient experienced the loss of sensation and does not respond to touch.

A triage nurse in a busy emergency department (ED) assesses a patient who complains of 7/10 abdominal pain and states, I had a temperature of 103.9 F (39.9 C) at home. The nurses first action should be to a. assess the patients current vital signs. b. give acetaminophen (Tylenol) per agency protocol. c. ask the patient to provide a clean-catch urine for urinalysis. d. tell the patient that it will 1 to 2 hours before being seen by the doctor.

a. The patients pain and statement about an elevated temperature indicate that the nurse should obtain vital signs before deciding how rapidly the patient should be seen by the health care provider. A urinalysis may be appropriate, but this would be done after the vital signs are taken. The nurse will not give acetaminophen before confirming a current temperature elevation.

An unresponsive 79-year-old is admitted to the emergency department (ED) during a summer heat wave. The patients core temperature is 105.4 F (40.8 C), blood pressure (BP) 88/50, and pulse 112. The nurse initially will plan to a. apply wet sheets and a fan to the patient. b. provide O2 at 6 L/min with a nasal cannula. c. start lactated Ringers solution at 1000 mL/hr. d. give acetaminophen (Tylenol) rectal suppository.

a. The priority intervention is to cool the patient. Antipyretics are not effective in decreasing temperature in heat stroke, and 100% oxygen should be given, which requires a high flow rate through a non-rebreather mask. An older patient would be at risk for developing complications such as pulmonary edema if given fluids at 1000 mL/hr.

Following an earthquake, patients are triaged by emergency medical personnel and are transported to the emergency department (ED). Which patient will the nurse need to assess first? a. A patient with a red tag b. A patient with a blue tag c. A patient with a black tag d. A patient with a yellow tag

a. The red tag indicates a patient with a life-threatening injury requiring rapid treatment. The other tags indicate patients with less urgent injuries or those who are likely to die.

After the return of spontaneous circulation following the resuscitation of a patient who had a cardiac arrest, therapeutic hypothermia is ordered. Which action will the nurse include in the plan of care? a. Apply external cooling device. b. Check mental status every 15 minutes. c. Avoid the use of sedative medications. d. Rewarm if temperature is <91 F (32.8 C).

a. When therapeutic hypothermia is used post-resuscitation, external cooling devices or cold normal saline infusions are used to rapidly lower body temperature to 89.6 F to 93.2 F (32 C to 34 C). Because hypothermia will decrease brain activity, assessing mental status every 15 minutes is not needed at this stage. Sedative medications are administered during therapeutic hypothermia.

The nurse is preparing to administer flumazenil to a patient having severe respiratory depression. For which reason is the nurse administering this antidote? a. morphine overdose b. lorazepam overdose c. ondansetron overdose d. promethazine overdose

b

The patient has a core temperature of 90 degree Fahrenheit. The most appropriate rewarming technique would be a. passive rewarming with warm blankets b. active internal rewarming using warmed IV fluids c. passive rewarming using air filled warming blankets d. active external rewarming by submerging in a warm bath

b

Which symptom would the nurse monitor in a patient who was given a benzo as an adjunct to general anesthesia? SATA a. insomnia b. hypotension c. tachycardia d. pulse oximetry e. abdominal distention

b, c Benzos can cause hypotension, tachycardia, and respiratory depression.

A person having an acute rejection of a transplanted kidney, what would help the nurse understand the course of events? (SATA) a. A new transplant should be considered b. acute rejection can be treated with OKT3 c. repeated episodes of acute rejection can lead to chronic rejection d. corticosteroids are the most successful drugs used to treat acute rejection e. acute rejection is common after a transplant and can be treated with drug therapy

b, c, e

Which factors can lead to a patient developing ascites? SATA a. Diabetes mellitus b. portal hypertension c. hyperaldosteronism d. decrease flow of hepatic lymph e. decrease serum colloidal oncotic pressure

b, c, e Portal hypertension causes an increase in resistance to blood flow in the liver leading to ascites. Hyperaldosteronism, or increased secretion of aldosterone, causes ascites. When there is decreased serum colloidal oncotic pressure, there is impairment of synthesis of albumin and loss of albumin in the peritoneal cavity, leading to ascites. Diabetes is a metabolic syndrome and does not cause ascites. Increased flow for hepatic lymph, not decreased flow, leads to ascites.

A patient who received a kidney transplant 30 days ago has a neutrophil count of 500/mm3. which rationales would the nurse use to explain the decreased neutrophil count for the patient? SATA a. Bacterial infection b. acute viral infection c. high creatinine levels d. Anti rejection medications e. symptoms of rejection by the kidney

b, d Rejection of the transplanted kidney does not cause leukopenia. Possible signs of rejection include decreased urine output, increased serum creatinine, hypertension, and edema. Immunosuppressants are given to prevent rejection; these drugs may also cause a reduction in the neutrophils, resulting in neutropenia. These drugs reduce immunity and make the patient susceptible for infection. Acute viral infections like cold or influenza can cause leukopenia. And increased WBC level is associated with bacterial infection. High creatinine levels do not cause leukopenia. However, increase creatinine levels are caused by kidney failure.

Important nursing interventions when caring for a patient with Cushing syndrome include (SATA) a. Restricting protein intake b. monitoring blood glucose levels c. observing for signs of hypotension d. administering medication to equal doses e. protecting patient from exposure to infection

b, e

When providing discharge instructions for a patient with a successful kidney transplant, which instructions would the nurse include to identify signs of organ rejection? SATA a. Notify the health care provider if there is a weight loss b. if your BP increases, call the office c. if you notice your urinary output increases, call someone d. notify the health care provider if your temperature goes below 98.6 degrees Fahrenheit e. if you're temperature exceeds 100 degree F, notify your provider right away

b, e Hypertension is a result of hypervolemia due to failure of the new kidney. The patient's body temperature will exceed 100 degrees F if a kidney is rejected. Weight gain will occur because the transplanted kidney fails to retain fluid. Urine output will be decreased or absent, depending on the degree of failure

Which victim of an explosion at an industrial plant would receive a black tag? a. Patient with chest pains after a crushing injury b. patient with a gaping head wound and no pulse c. patient with massive bleeding from the lower leg patient with weak, thready pulses and hypotension

b. A patient with a gaping head wound and no pulse is dead and should receive a black tag. Patients having chest pains after a crushing injury, or massive bleeding or weak pulse and hypotension has a life threatening injury and would receive a red tag.

The nurse is caring for a 73-year-old man who has cirrhosis. Which data obtained by the nurse during the assessment will be of most concern? a. The patient complains of right upper-quadrant pain with palpation. b. The patients hands flap back and forth when the arms are extended. c. The patient has ascites and a 2-kg weight gain from the previous day. d. The patients skin has multiple spider-shaped blood vessels on the abdomen.

b. Asterixis indicates that the patient has hepatic encephalopathy, and hepatic coma may occur. The spider angiomas and right upper quadrant abdominal pain are not unusual for the patient with cirrhosis and do not require a change in treatment. The ascites and weight gain indicate the need for treatment but not as urgently as the changes in neurologic status.

Which assessment finding would the nurse expect in the patient with a high ammonia level associated with hepatic encephalopathy? a. Aphasia b. Asterixis c. Hyperactivity d. acute dementia

b. Asterixis is a twitching spasm of the hands and wrist in patients with increased ammonia levels and conditions such as hepatic encephalopathy.

Which patient seen in the ED after exposure to severe cold temperatures require the most rapid action by the nurse? a. Patient with severely painful, cold, pale fingertips b. patient with pale, firm feet that are insensitive to touch c. patient with temperature 98 degrees F and shivering d. patient with edema and blisters on the lower leg and foot

b. Pallor, firmness, and insensitivity to touch of an extremity after exposure to cold indicate deep frostbite, which will require immediate rewarming in order to restore circulation to the patient's feet and avoid necrosis and amputation.

To detect possible complications in a patient with severe cirrhosis who has bleeding esophageal varices, it is most important for the nurse to monitor a. bilirubin levels. b. ammonia levels. c. potassium levels. d. prothrombin time.

b. The protein in the blood in the gastrointestinal (GI) tract will be absorbed and may result in an increase in the ammonia level because the liver cannot metabolize protein very well. The prothrombin time, bilirubin, and potassium levels should also be monitored, but they will not be affected by the bleeding episode.

The nurse is providing support for the family of a patient who died immediately after arrival in the emergency department. Which intervention by the nurse would be most supportive of the families spiritual needs? a. Quietly escort the family to the hospital chapel and staying with them b. Ask the family whether they would like a visit from the hospital chaplain, who is nearby c. encouraged a family to spend time with their loved one to allow them to begin to accept that death as real d. review the patient's medical record to determine religious affiliation and contact the appropriate entity

b. When a family suffers a loss in the ED, the nurse can support spiritual needs by asking whether they would like to see the hospital chaplain, who is often readily available. The family may not find a visit to the chapel helpful. Assuring the family that all measures were taken does not address their spiritual needs. While encouraging the family to be with the loved one may help them to accept the loss, it will not specifically address their spiritual needs. Religious affiliation may or may not be available in the electronic health record; religion reflects formal practices, whereas spirituality reflects the meaning one attaches to life.

For which clinical manifestation would the nurse monitor and patient experiencing chronic organ rejection after a renal transplant? a. Loss of bile duct b. Glomerulopathy c. bronchiolitis obliterans d. accelerated coronary artery disease

b. Chronic rejection occurs when a transplanted organ is infiltrated with large numbers of T&B cells. Chronic rejection after KIDNEY transplantation often manifest as glomerulopathy. Chronic rejection after LIVER transplantation manifests as loss of bile duct. Chronic rejection after LUNG transplantation is indicated by bronchiolitis obliterans. Chronic rejection after HEART transplantation is indicated by accelerated coronary artery disease.

Which color tag will the nurse use after assessing a patient with a pulse of 20 and a severe skull fracture at the site of a mass casualty incident? a. Black b. Blue c. Green d. Yellow

b. Since the patient is unlikely to survive, the nurse will use a blue tag. A black tag is used to identify patients who are dead.

Which effect may be observed if large amounts of endogenous corticosteroids are released into systemic circulation during surgery on a patient with Cushing syndrome? a. Fatigue b. Infections c. Delusions d. Hypotension

b. A patient may become susceptible to infections if the endogenous corticosteroid levels are high during surgery. Hypertension, not hypotension, is observed due to increased levels of corticosteroids.

During the primary assessment of a victim of a motor vehicle collision, the nurse determines that the patient is breathing and has an unobstructed airway. Which action should the nurse take next? a. Palpate extremities for bilateral pulses. b. Observe the patients respiratory effort. c. Check the patients level of consciousness. d. Examine the patient for any external bleeding.

b. Even with a patent airway, patients can have other problems that compromise ventilation, so the next action is to assess the patients breathing. The other actions are also part of the initial survey but assessment of breathing should be done immediately after assessing for airway patency.

Which action should the nurse take to evaluate treatment effectiveness for a patient who has hepatic encephalopathy? a. Request that the patient stand on one foot. b. Ask the patient to extend both arms forward. c. Request that the patient walk with eyes closed. d. Ask the patient to perform the Valsalva maneuver.

b. Extending the arms allows the nurse to check for asterixis, a classic sign of hepatic encephalopathy. The other tests might also be done as part of the neurologic assessment but would not be diagnostic for hepatic encephalopathy.

A patient who has experienced blunt abdominal trauma during a motor vehicle collision is complaining of increasing abdominal pain. The nurse will plan to teach the patient about the purpose of a. peritoneal lavage. b. abdominal ultrasonography. c. nasogastric (NG) tube placement. d. magnetic resonance imaging (MRI).

b. For patients who are at risk for intraabdominal bleeding, focused abdominal ultrasonography is the preferred method to assess for intraperitoneal bleeding. An MRI would not be used. Peritoneal lavage is an alternative, but it is more invasive. An NG tube would not be helpful in diagnosis of intraabdominal bleeding.

Gastric lavage and administration of activated charcoal are ordered for an unconscious patient who has been admitted to the emergency department (ED) after ingesting 30 lorazepam (Ativan) tablets. Which action should the nurse plan to do first? a. Insert a large-bore orogastric tube. b. Assist with intubation of the patient. c. Prepare a 60-mL syringe with saline. d. Give first dose of activated charcoal.

b. In an unresponsive patient, intubation is done before gastric lavage and activated charcoal administration to prevent aspiration. The other actions will be implemented after intubation.

For a patient with cirrhosis, which of the following nursing actions can the registered nurse (RN) delegate to unlicensed assistive personnel (UAP)? a. Assessing the patient for jaundice b. Providing oral hygiene after a meal c. Palpating the abdomen for distention d. Assisting the patient to choose the diet

b. Providing oral hygiene is within the scope of UAP. Assessments and assisting patients to choose therapeutic diets are nursing actions that require higher-level nursing education and scope of practice and would be delegated to licensed practical/vocational nurses (LPNs/LVNs) or RNs.

A serum potassium level of 3.2 mEq/L (3.2 mmol/L) is reported for a patient with cirrhosis who has scheduled doses of spironolactone (Aldactone) and furosemide (Lasix). due. Which action should the nurse take? a. Administer both drugs. b. Administer the spironolactone. c. Withhold the spironolactone and administer the furosemide. d. Withhold both drugs until discussed with the health care provider.

b. Spironolactone is a potassium-sparing diuretic and will help increase the patients potassium level. The nurse does not need to talk with the doctor before giving the spironolactone, although the health care provider should be notified about the low potassium value. The furosemide will further decrease the patients potassium level and should be held until the nurse talks with the health care provider.

Which assessment information will be most important for the nurse to report to the health care provider about a patient with acute cholecystitis? a. The patients urine is bright yellow. b. The patients stools are tan colored. c. The patient has increased pain after eating. d. The patient complains of chronic heartburn.

b. Tan or grey stools indicate biliary obstruction, which requires rapid intervention to resolve. The other data are not unusual for a patient with this diagnosis, although the nurse would also report the other assessment information to the health care provider.

A patient who is unconscious after a fall from a ladder is transported to the emergency department by emergency medical personnel. During the primary survey of the patient, the nurse should a. obtain a complete set of vital signs. b. obtain a Glasgow Coma Scale score. c. ask about chronic medical conditions. d. attach a cardiac electrocardiogram monitor.

b. The Glasgow Coma Scale is included when assessing for disability during the primary survey. The other information is part of the secondary survey.

Which topic is most important to include in patient teaching for a 41-year-old patient diagnosed with early alcoholic cirrhosis? a. Maintaining good nutrition b. Avoiding alcohol ingestion c. Taking lactulose (Cephulac) d. Using vitamin B supplements

b. The disease progression can be stopped or reversed by alcohol abstinence. The other interventions may be used when cirrhosis becomes more severe to decrease symptoms or complications, but the priority for this patient is to stop the progression of the disease.

A patient is admitted to the emergency department (ED) after falling through the ice while ice skating. Which assessment will the nurse obtain first? a. Heart rate b. Breath sounds c. Body temperature d. Level of consciousness

b. The priority assessment relates to ABCs (airway, breathing, circulation) and how well the patient is oxygenating, so breath sounds should be assessed first. The other data will also be collected rapidly but are not as essential as the breath sounds.

The patient admitted to the hospital with cirrhosis of the liver suddenly vomits blood. Which action will the nurse take? a. Give Propranolol orally b. check for signs of ascites c. stabilize the patient and manage the airway d. prepare the patient for a shunting procedure

c

Which physiological factor is related to the development of Cushing syndrome? a. Liver dysfunction b. chronic renal failure c. excessive secretion of ACTH d. decrease secretion of ACTH

c

The following four patients arrive in the emergency department (ED) after a motor vehicle collision. In which order should the nurse assess them? a. A 74-year-old with palpitations and chest pain b. A 43-year-old complaining of 7/10 abdominal pain c. A 21-year-old with multiple fractures of the face and jaw d. A 37-year-old with a misaligned left leg with intact pulses

c, a, b, d The highest priority is to assess the 21-year-old patient for airway obstruction, which is the most life- threatening injury. The 74-year-old patient may have chest pain from cardiac ischemia and should be assessed and have diagnostic testing for this pain. The 43-year-old patient may have abdominal trauma or bleeding and should be seen next to assess circulatory status. The 37-year-old appears to have a possible fracture of the left leg and should be seen soon, but this patient has the least life-threatening injury.

Which topics would the nurse discuss with the patient in preparation for a scheduled kidney transplant? SATA a. Avoid all exercise, work, and sports activities b. production of urine will be delayed after surgery c. immunosuppressive drugs will be required daily d. transplantation will be performed only if crossmatching is negative e. symptoms of rejection include a decrease in temperature and BP

c, d Immunosuppressive agents are administered daily to reduce the immune systems tendency to reject the transplanted organ. A crossmatch uses serum from the recipient mixed with donor lymphocytes to test for any preformed anti-human leukocyte antigen (HLA) antibodies to the potential donor organ. A negative crossmatch indicates that no preformed antibodies are present, and it is safe to proceed with transplantation. Urine production occurs almost immediately. If the transplant is rejected, the patient may experience a rise in temperature and BP due to fluid retention. Although recreation and exercise are encouraged, strenuous sports activities should be strictly avoided.

A nurse is caring for a patient who has cirrhosis of the liver. Which clinical manifestations would the nurse expect to find? SATA a. White patches on the skin b. deposits of dark pigments c. small areas of bleeding into the skin d. vascular lesions formed by small blood vessels e. small dilated blood vessels with spider like branches

c, d, e

The emergency department (ED) nurse is initiating therapeutic hypothermia in a patient who has been resuscitated after a cardiac arrest. Which actions in the hypothermia protocol can be delegated to an experienced licensed practical/vocational nurse (LPN/LVN) (select all that apply)? a. Continuously monitor heart rhythm. b. Check neurologic status every 2 hours. c. Place cooling blankets above and below patient. d. Give acetaminophen (Tylenol) 650 mg per nasogastric tube. e. Insert rectal temperature probe and attach to cooling blanket control panel.

c, d, e Experienced LPN/LVNs have the education and scope of practice to implement hypothermia measures (e.g., cooling blanket, temperature probe) and administer medications under the supervision of a registered nurse (RN). Assessment of neurologic status and monitoring the heart rhythm require RN-level education and scope of practice and should be done by the RN.

Which risk factor is most commonly associated with cirrhosis? a. Hepatitis A b. Polypharmacy c. alcohol abuse d. IV drug abuse

c.

The patient with cirrhosis is learning self-care. Which statement indicates that the patient needs more teaching? a. "Scrotum support may be more comfortable when I have scrotal edema." b. "I need to take good care of my belly and ankle skin where it is swollen." c. "If I notice a fast heart rate or irregular beats, that is normal for cirrhosis." d. "I can use pillows to support my head to help me breathe when I am in bed."

c. If the patient would cirrhosis experiences a fast irregular heart rate, it may be indicative of hypokalemia and should be reported to HCP because this is not normal for cirrhosis and it is potentially dangerous.

Which intervention does the nurse implement for the active external rewarming of a patient experiencing hypothermia? a. Administering humidified oxygen b. remove the patient's wet clothing c. immerse the patient in warm water d. place the patient under radiant lighting

c. Immersing the patient in warm water is an intervention for active external rewarming. Administering humidified oxygen is an intervention for active internal rewarming. Interventions for passive rewarming include removing the patient's wet clothing and placing the patient under radiant lighting.

After the nurse examines the trust and upper body of a patient after a motor vehicle accident, which finding is most important to report quickly to the health care provider? a. A bruise on the right chest b. lacerations along the arms and legs c. muffled and low pitch heart sounds d. complaints of pain when the chest is touched

c. Muffled heart tones may indicate trauma to the heart, and the patient should be immediately evaluated for cardiac injury caused by blunt chest trauma.

The patient is admitted to the emergency department after falling from a roof. To which assessment finding does the nurse assign the highest priority? a. Pale, moist skin b. pain on inspiration c. asymmetrical chest movement d. laceration on a lower extremity

c. Prioritization of care for this patient follows an initial assessment of ABC and neurological status. In a traumatic injury, ribs may be fractured and detached from the chest wall, resulting in asymmetric chest movement. This may compromise respiratory status and indicate a pneumothorax or other internal injuries. After identification of asymmetric chest movement, pain on inspiration would be evaluated as part of the respiratory assessment. After the breathing and respiratory issues are assessed, pale, moist skin and lower extremity lacerations may be evaluated as part of the circulatory assessment.

To prepare a 56-year-old male patient with ascites for paracentesis, the nurse a. places the patient on NPO status. b. assists the patient to lie flat in bed. c. asks the patient to empty the bladder. d. positions the patient on the right side.

c. The patient should empty the bladder to decrease the risk of bladder perforation during the procedure. The patient would be positioned in Fowlers position and would not be able to lie flat without compromising breathing. Because no sedation is required for paracentesis, the patient does not need to be NPO.

A 38-year-old patient with cirrhosis has ascites and 4+ edema of the feet and legs. Which nursing action will be included in the plan of care? a. Restrict daily dietary protein intake. b. Reposition the patient every 4 hours. c. Place the patient on a pressure-relieving mattress. d. Perform passive range of motion daily.

c. The pressure-relieving mattress will decrease the risk for skin breakdown for this patient. Adequate dietary protein intake is necessary in patients with ascites to improve oncotic pressure. Repositioning the patient every 4 hours will not be adequate to maintain skin integrity. Passive range of motion will not take the pressure off areas such as the sacrum that are vulnerable to breakdown.

The nurse assess that a patient who sustained a severe trauma injury has asymmetric chest while movement and no breath sounds on one side of the chest. The nurse identifies that which treatment strategy will help to alleviate the patient's symptoms? a. Administering mannitol b. administering chlorpromazine c. applying a bag valve mask BVM d. Infusing warmed IV fluids

c. When a patient has asymmetric chest wall movement and no breath sounds on one side of the chest, it indicates that the patient has a flow chest or a pneumothorax. This means the patient may have respiratory distress. Therefore the nurse should provide ventilation to the patient by using a BVM for effective treatment.

Which action does the nurse take for a patient with hyperacute rejection 6 hours after a kidney transplant surgery? a. Administer antibiotics b. transfer to the cardiac catheterization lab c. prepare the patient for the operating room d. anticipate an increase in immunosuppressive therapy

c. A patient with hyperacute rejection of a recent kidney transplant would return to the operating room for removal of the kidney. Antibiotics are given to treat infection in a transplant patient. Cardiac catheterization may be indicated for chronic rejection of the transplant. An increase in immunosuppression would be indicated for a patient with acute transplant rejection.

Which describes the primary purpose of administering 100% oxygen to a patient with heat stroke? a. To prevent dysrhythmias b. to increase cerebral blood flow c. to compensate for a hypermetabolic state d. to correct the effects of an electrolyte imbalance

c. Administering 100% oxygen to a patient with heat stroke compensates for the hypermetabolic state. The primary purpose of administering oxygen is not to prevent dysrhythmias. The oxygen does not increase cerebral blood flow or correct the effects of electrolyte imbalance.

The nurse is conducting a secondary survey for a patient involved in a motor vehicle accident. Which initial action does the nurse take after moving the patient's injured extremity? a. Elevate the extremity b. immobilize the extremity c. assess the pulse of the extremity d. apply ice packs to the injured area

c. After movement of an injured extremity, the nurse will assess the pulse to ensure the extremity is receiving circulation. Elevation, immobilization, and applying ice packs to the injured area are not initial actions the nurse takes after moving the extremity.

During change-of-shift report, the nurse learns about the following four patients. Which patient requires assessment first? a. 40-year-old with chronic pancreatitis who has gnawing abdominal pain b. 58-year-old who has compensated cirrhosis and is complaining of anorexia c. 55-year-old with cirrhosis and ascites who has an oral temperature of 102 F (38.8 C) d. 36-year-old recovering from a laparoscopic cholecystectomy who has severe shoulder pain

c. This patients history and fever suggest possible spontaneous bacterial peritonitis, which would require rapid assessment and interventions such as antibiotic therapy. The clinical manifestations for the other patients are consistent with their diagnoses and do not indicate complications are occurring.

The nurse is caring for a patient following an adrenalectomy. The highest priority in the immediate postoperative period is to a. protect the patients skin. b. monitor for signs of infection. c. balance fluids and electrolytes. d. prevent emotional disturbances.

c. After adrenalectomy, the patient is at risk for circulatory instability caused by fluctuating hormone levels, and the focus of care is to assess and maintain fluid and electrolyte status through the use of IV fluids and corticosteroids. The other goals are also important for the patient but are not as immediately life threatening as the circulatory collapse that can occur with fluid and electrolyte disturbances.

Family members are in the patients room when the patient has a cardiac arrest and the staff start resuscitation measures. Which action should the nurse take next? a. Keep the family in the room and assign a staff member to explain the care given and answer questions. b. Ask the family to wait outside the patients room with a designated staff member to provide emotional support. c. Ask the family members about whether they would prefer to remain in the patients room or wait outside the room. d. Tell the family members that patients are comforted by having family members present during resuscitation efforts.

c. Although many family members and patients report benefits from family presence during resuscitation efforts, the nurses initial action should be to determine the preference of these family members. The other actions may be appropriate, but this will depend on what is learned when assessing family preferences.

A 22-year-old patient who experienced a near drowning accident in a local pool, but now is awake and breathing spontaneously, is admitted for observation. Which assessment will be most important for the nurse to take during the observation period? a. Auscultate heart sounds. b. Palpate peripheral pulses. c. Auscultate breath sounds. d. Check pupil reaction to light.

c. Because pulmonary edema is a common complication after near drowning, the nurse should assess the breath sounds frequently. The other information also will be obtained by the nurse, but it is not as pertinent to the patients admission diagnosis.

A 49-year-old female patient with cirrhosis and esophageal varices has a new prescription for propranolol (Inderal). Which finding is the best indicator that the medication has been effective? a. The patient reports no chest pain. b. Blood pressure is 140/90 mm Hg. c. Stools test negative for occult blood. d. The apical pulse rate is 68 beats/minute.

c. Because the purpose of b-blocker therapy for patients with esophageal varices is to decrease the risk for bleeding from esophageal varices, the best indicator of the effectiveness for propranolol is the lack of blood in the stools. Although propranolol is used to treat hypertension, angina, and tachycardia, the purpose for use in this patient is to decrease the risk for bleeding from esophageal varices.

A patient with hypotension and an elevated temperature after working outside on a hot day is treated in the emergency department (ED). The nurse determines that discharge teaching has been effective when the patient makes which statement? a. I will take salt tablets when I work outdoors in the summer. b. I should take acetaminophen (Tylenol) if I start to feel too warm. c. I should drink sports drinks when working outside in hot weather. d. I will move to a cool environment if I notice that I am feeling confused.

c. Electrolyte solutions such as sports drinks help replace fluid and electrolytes lost when exercising in hot weather. Salt tablets are not recommended because of the risks of gastric irritation and hypernatremia. Antipyretic medications are not effective in lowering body temperature elevations caused by excessive exposure to heat. A patient who is confused is likely to have more severe hyperthermia and will be unable to remember to take appropriate action.

A nurse has obtained donor tissue typing information about a patient who is waiting for a kidney transplant. Which results should be reported to the transplant surgeon? a. Patient is Rh positive and donor is Rh negative b. Six antigen matches are present in HLA typing c. Results of patient-donor cross matching are positive d. Panel of reactive antibodies (PRA) percentage is low

c. Positive crossmatching is an absolute contraindication to kidney transplantation, since a hyperacute rejection will occur after the transplant. The other information indicates that the tissue match between the patient and potential donor is acceptable.

A 37-year-old patient is being admitted with a diagnosis of Cushing syndrome. Which findings will the nurse expect during the assessment? a. Chronically low blood pressure b. Bronzed appearance of the skin c. Purplish streaks on the abdomen d. Decreased axillary and pubic hair

c. Purplish-red striae on the abdomen are a common clinical manifestation of Cushing syndrome. Hypotension and bronzed-appearing skin are manifestations of Addisons disease. Decreased axillary and pubic hair occur with androgen deficiency.

Which information obtained by the nurse in the endocrine clinic about a patient who has been taking prednisone (Deltasone) 40 mg daily for 3 weeks is most important to report to the health care provider? a. Patients blood pressure is 148/94 mm Hg. b. Patient has bilateral 2+ pitting ankle edema. c. Patient stopped taking the medication 2 days ago. d. Patient has not been taking the prescribed vitamin D.

c. Sudden cessation of corticosteroids after taking the medication for a week or more can lead to adrenal insufficiency, with problems such as severe hypotension and hypoglycemia. The patient will need immediate evaluation by the health care provider to prevent and/or treat adrenal insufficiency. The other information will also be reported, but does not require rapid treatment.

Which assessment finding of a 42-year-old patient who had a bilateral adrenalectomy requires the most rapid action by the nurse? a. The blood glucose is 176 mg/dL. b. The lungs have bibasilar crackles. c. The blood pressure (BP) is 88/50 mm Hg. d. The patient reports 5/10 incisional pain.

c. The decreased BP indicates possible adrenal insufficiency. The nurse should immediately notify the health care provider so that corticosteroid medications can be administered. The nurse should also address the elevated glucose, incisional pain, and crackles with appropriate collaborative or nursing actions, but prevention and treatment of acute adrenal insufficiency is the priority after adrenalectomy.

The charge nurse is assigning rooms for new admissions. Which patient would be the most appropriate roommate for a patient who has acute rejection of an organ transplant? a. A patient who has viral pneumonia b. A patient with second-degree burns c. A patient who is recovering from an anaphylactic reaction to a bee sting d. A patient with graft-versus-host disease after a recent bone marrow transplant

c. Treatment for a patient with acute rejection includes administration of additional immunosuppressants, and the patient should not be exposed to increased risk for infection as would occur from patients with viral pneumonia, graft-versus-host disease, and burns. There is no increased exposure to infection from a patient who had an anaphylactic reaction.

A patient has been taking prednisone for several weeks after experiencing a hypersensitivity reaction. To prevent adrenal insufficiency, the nurse should ensure that the patient knows to do which of the following? A) Take the drug concurrent with levothyroxine (Synthroid) B) Take each dose of prednisone with a dose of calcium C) Gradually replace the prednisone with an OTC alternative D) Slowly taper down the dose of prednisone, as ordered

d

The HCP prescribes Lactulose for a patient with hepatic encephalopathy. Which data will the nurse monitor for effectiveness of this medication? a. Liver enzymes b. stool frequency c. abdominal pain d. ammonia levels

d

What accurately describes rejection after transplantation? a. Hyperacute rejection can be treated with OKT3 b. Acute rejection can be treated with sirolimus or tacrolimus c. chronic rejection can be treated with tacrolimus or cyclosporine d. hyperacute reaction can be avoided if cross matching is done before transplantation

d

Which disorder is characterized by fat pads on the back of the neck, and increase abdominal girth, a Buffalo hump, and a moon face? a. Acromegaly b. Conn's disease c. Graves' disease d. Cushing syndrome

d

Which action will the nurse take first to assess LOC in a patient who has just been transported to the emergency department after an accident? a. Touch the patient b. apply a painful stimulus c. call the patient by name d. observe the patient

d. The initial assessment is observation of the patient for indications of alertness such as open eyes and looking around the exam room. If the patient's eyes are closed, the next action would be to call the patient by name and assess the patient's response. Next the nurse would touch the patient and assess the patient's response to touch. If there is no response to touch, the nurse will apply a painful stimulus such as nail bed pressure and assess whether the patient responds.

Which data will the nurse monitor in relation to the 4+ pitting edema assessed in a patient with cirrhosis? a. Hemoglobin b. Temperature c. Activity level d. Albumin level

d. The low oncotic pressure caused by hypoalbuminemia is a major pathophysiologic factor in the development of edema. The other parameters should also be monitored, but they are not directly associated with the patients current symptoms.

Which clinical manifestation will the nurse expect when caring for a patient who was exposed to cold weather and has a temperature of 91 degree F? a. Peripheral edema b. flaccid skeletal muscles c. fixed and dilated pupils d. heart rate 46 beats/minute

d. With moderate hypothermia, patients develop bradycardia. Skeletal muscles become more rigid with hypothermia. Pupils are fixed and dilated with severe hypothermia.

A patient is stable after treatment and recently diagnosed esophageal varices. Which information would the nurse include in the teaching plan for this patient? a. Decrease fluid intake to avoid irritating the varices b. eat foods quickly so they do not get cold and cause distress c. avoid exercise because it may cause bleeding of the varices d. avoid straining during defecation to keep venous pressure low

d. straining during a bowel movement increases venous pressure and could cause rupture of the varices.

Which assessment finding would the nurse expect from caring for a patient who has gallstones? a. Tachycardia b. Hypertension c. Hyperglycemia d. clay colored stools

d. the presence of gallstones results in bile flow obstruction, which is manifested by fever, clay colored stools and dark urine.

When triaging for a mash casualty incident, which color tag will the nurse use after assessing a patient whose condition is not life threatening but who has multiple injuries? a. Red b. Blue c. Green d. Yellow

d. Yellow-colored tags are used for patients who need urgent medical attention but whose condition is not life threatening. Red tags are used to indicate life-threatening conditions that need immediate interventions. Blue tags are used for patients who are expected to die. Green tags are used for patients with minor injuries.

When rewarming a patient who arrived in the emergency department (ED) with a temperature of 87 F (30.6 C), which assessment indicates that the nurse should discontinue active rewarming? a. The patient begins to shiver. b. The BP decreases to 86/42 mm Hg. c. The patient develops atrial fibrillation. d. The core temperature is 94 F (34.4 C).

d. A core temperature of 89.6 F to 93.2 F (32 C to 34 C) indicates that sufficient rewarming has occurred. Dysrhythmias, hypotension, and shivering may occur during rewarming and should be treated but are not an indication to stop rewarming the patient.

An older adult patient has a prescription for cyclosporine following a kidney transplant. Which information in the patients health history has the most implications for planning patient teaching about the medication at this time? a. The patient restricts salt to treat prehypertension. b. The patient drinks 3 to 4 quarts of fluids every day. c. The patient has many concerns about the effects of cyclosporine. d. The patient has a glass of grapefruit juice every day for breakfast.

d. Grapefruit juice can increase the toxicity of cyclosporine. The patient should be taught to avoid grapefruit juice. High fluid intake will not affect cyclosporine levels or renal function. Cyclosporine may cause hypertension, and the patients many concerns should be addressed, but these are not potentially life-threatening problems.

The emergency department (ED) triage nurse is assessing four victims involved in a motor vehicle collision. Which patient has the highest priority for treatment? a. A patient with no pedal pulses. b. A patient with an open femur fracture. c. A patient with bleeding facial lacerations. d. A patient with paradoxic chest movements.

d. Most immediate deaths from trauma occur because of problems with ventilation, so the patient with paradoxic chest movements should be treated first. Face and head fractures can obstruct the airway, but the patient with facial injuries only has lacerations. The other two patients also need rapid intervention but do not have airway or breathing problems.

During the primary survey of a patient with severe leg trauma, the nurse observes that the patients left pedal pulse is absent and the leg is swollen. Which action will the nurse take next? a. Send blood to the lab for a complete blood count. b. Assess further for a cause of the decreased circulation. c. Finish the airway, breathing, circulation, disability survey. d. Start normal saline fluid infusion with a large-bore IV line.

d. The assessment data indicate that the patient may have arterial trauma and hemorrhage. When a possibly life-threatening injury is found during the primary survey, the nurse should immediately start interventions before proceeding with the survey. Although a complete blood count is indicated, administration of IV fluids should be started first. Completion of the primary survey and further assessment should be completed after the IV fluids are initiated.

Which clinical manifestation would be most prominent in the assessment of a patient with suspected Cushing syndrome? a. Hypertension with periods of dizziness b. bulking up of skeletal muscle c. hypoglycemia with intense hunger d. weight gain, including truncal obesity

d. The most prominent clinical manifestation in Cushing syndrome is weight gain leading to truncal obesity with characteristic rounded moon phase and fat deposits in the neck and upper back also known as Buffalo hump.

A 44-year-old female patient with Cushing syndrome is admitted for adrenalectomy. Which intervention by the nurse will be most helpful for a nursing diagnosis of disturbed body image related to changes in appearance? a. Reassure the patient that the physical changes are very common in patients with Cushing syndrome. b. Discuss the use of diet and exercise in controlling the weight gain associated with Cushing syndrome. c. Teach the patient that the metabolic impact of Cushing syndrome is of more importance than appearance. d. Remind the patient that most of the physical changes caused by Cushing syndrome will resolve after surgery.

d. The most reassuring communication to the patient is that the physical and emotional changes caused by the Cushing syndrome will resolve after hormone levels return to normal postoperatively. Reassurance that the physical changes are expected or that there are more serious physiologic problems associated with Cushing syndrome are not therapeutic responses. The patients physiological changes are caused by the high hormone levels, not by the patients diet or exercise choices.

A nurse, answering a patient's callbell, notices upon entering the patient's room that the patient is sweating profusely, pale, and anxious. The patient states, "I feel really dizzy and my vision is blurred." Which intervention is the highest priority? a. Obtain vital signs b. administer Glucagon c. have the patient lie flat d. check blood glucose level

d

Define diabetes mellitus

Most often referred to as diabetes, is a chronic multisystem disease characterized by hyperglycemia from abnormal insulin production, impaired insulin use, or both

Nursing Intervention for DKA/HHS

1) Fluid Resuscitation a. 0.45-0.9% NS UNTIL blood glucose reaches 250 mg/dL. Then add 5-10% dextrose to prevent sudden drop of BG (sudden hypoglycemia can cause cerebral edema) b. Monitor for S/S of fluid overload 2) Insulin therapy. Check potassium levels PRIOR to giving insulin -0.1 units/kg/hr 3) Potassium Electrolyte

Precipitating factors of DKA

1) Illness, infection 2) inadequate insulin dosage 3) undiagnosed type one diabetes 4) lack of education, understanding, or resources, and neglect

Clinical Manifestations of DKA

1) Kussmaul respiration - deep, rapid breathing 2) fruity breath 3) abdominal pain 4) serum glucose >250mg/dL, pH <7.30, HCO3 < 16 mEq/L 5) glucose and ketones in urine 6) S/S of dehydration Diagnose DKA: BG, ketones in urine and metabolic acidosis

Nursing Intervention of Addison's Disease

1) Lifelong Hormone Therapy - hydrocortisone (increase during periods of stress) -fludrocortisone Androgen replacement (for women) 2) increase salt intake 3) Tx for Addisonian Crisis: - shock management - high dose of hydrocortisone replacement - large volume 0.9% NS and 5% dextrose

Nursing Interventions for Hypoglycemia

1) check BG and treat with Rule of 15 if <70 mg/dL -15g of carbs (4-6oz of juice/regular soft drink, 1 TBSP of honey/syrup, 4 TSP of jelly) -wait 15 min and recheck BG -another 15g of carbs if <70mg/dL (cheese, crackers, peanut butter crackers. AVOID: candy bars, cookies, whole milk, ice cream) -if BG is stable, give additional carbs if next meal is 1 hr away or engaged in physical activity -call HCP or EMS if unsuccessful 2) Acute care with unresponsive patient -IV D50 or IM glucagon (if IV not available) -S/E nausea: turn client to the side

Define Addison's Disease

Adrenocortical insufficiency (hypofunction of adrenal cortex = ↓ glucocorticoids, mineralocorticoids, and androgens) Causes: 1) 80% caused by autoimmune response 2) TB, AIDS, fungal infections, metastatic cancer 3) adrenal hemorrhage, chemo, therapy for AIDS, bilateral adrenalectomy

A follow up visit to learn about management of hypoglycemia, a patient's blood glucose level was found to be 65 mg/dL. Which action by the patient might be responsible for this? a. Including whole milk in the diet b. including more fruits and juices in the diet c. performing moderate amounts of exercise d. including large quantities of rapid acting carbs in the diet

a. Whole milk contains a high fat content and will reduce glucose absorption, and it may cause hypoglycemia.

Which clinical manifestation is seen with Addison's disease? a. Delusions b. Hypokalemia c. Hyperglycemia d. truncal obesity

a. Addison's disease occurs due to the hypofunction of adrenal cortex. This hypofunction manifests as delusions, which occur due to decreased levels of glucocorticoids. Hypokalemia, hyperglycemia, and truncal obesity are clinical manifestations of Cushing syndrome that occur due to the hyperfunction of the adrenal cortex.

A patient with type 1 DM calls the clinic reporting nausea, vomiting and diarrhea. It is most important that the nurse advise the patient to a. withhold the regular dose of insulin b. drink cool fluids with high glucose content c. check BG level every 2-4 hours d. use a less strenuous form of exercise than usual until illness resolves

c

Which statement by the patient with type 2 diabetes is accurate? a. "I will limit my alcohol intake to one drink each day." b. "I am not allowed to eat any sweets because of my diabetes." c. "I cannot exercise because I take a blood glucose lowering medication." d. "The amount of fat in my diet is not important only carbs raise my blood sugar."

a

Which are the appropriate therapies for a patients with diabetes? Select all that apply a. use of statins to reduce CVD risk b. use of diuretics to treat nephropathy c. use of ACE inhibitors to treat neuropathy d. use of serotonin agonist to decrease appetite e. use of laser photocoagulation to treat retinopathy

a, c, e

Nursing Intervention for SIADH

Treatment: correct fluid volume excess 1) Mild hyponatremia (>125 mEq/L) - fluid restriction (800-1000 mL/day) - furosemide - demeclocycline (block ADH effects) 2) severe hyponatremia (<120 mEq/L) - fluid restriction (500 mL/day) -small amount of IV hypertonic saline solution (3% NS) to be infused SLOWLY -vasopressors receptor antagonists (block ADH activity): Conivaptan (Vaprisol) and Tolvaptan (Samsca) - contraindicated with liver failure!! 3) Chronic SIADH - restrict fluid 800-1000 mL/day - ice chips/chew sugarless gum to decrease thirst Seizure and fall precautions, HOB <10 degree to promote venous return to heart, daily wts, Na+ and K diet supplements, S/S of electrolyte imbalance

You are caring for a patient with newly diagnosed type one diabetes. What information is essential to include in your patient teaching before discharge from the hospital? (select all that apply) a. Insulin administration b. elimination of sugar from diet c. need to reduce physical activity d. use of portable blood glucose monitor e. hypoglycemia prevention, symptoms, and treatment

a, d, e

What may lead to a coma in patients with DKA if left untreated? (SATA) a. Dehydration b. Hypoglycemia c. cerebral edema d. metabolic acidosis e. electrolyte imbalance

a, d, e dehydration, metabolic acidosis, and electrolyte imbalance may lead to a coma if untreated. Hypoglycemia and cerebral edema are not associated with coma in patients with DKA

Diagnose and Treat DI

Dx: Water Deprivation Test -Pt. deprived of water for 8-12 hrs or until 3%-5% of body wt. is lost, then given desmopressin acetate (DDAVP) -Test terminated if tachycardia, excessive wt. loss, or hypotension develops. Consequently, BP and HR monitoring are priorities during testing therapy Tx: fluid and hormone - IV hypotonic saline or dextrose 5% (titrated) -Hormone DDAVP (Desmopressin): analog of ADH (If given too much can result in SIADH, assess for S/S) -Drug Vasopressin: ADH (monitor HR, BP, LOC, I/O, specific gravity) -Carbamazepine (Tegretol): decrease thirst

S/S of Addison's Disease

Slow onset: 1) muscular weakness, fatigue, hypotension 2) Nausea, diarrhea, abdominal pain, anorexia 3) hyper/dark pigmentation of skin 4) salt craving Complication: 1) Addisonian crisis (life-threatening emergency) -acute adrenal insufficiency caused by sudden sharp drop in hormones -fever, weak, confused, severe vomiting/diarrhea, pain -hypotension, tachy, dehydration, shock -low sodium and glucose, high potassium

Define SIADH

Too much ADH (reabsorption of water = fluid retention) 1) ECF expands 2) ↓ serum osmolality (<280 mOms/kg) 3) ↓ Na < 135 mEq/L (dilutional hyponatremia) and hypochloremia 4) concentrated urine (urine specific gravity >1.030) 5) ↑ GFR Causes: 1) small cell lung cancer, chemo agents (Vincristine) 2) brain tumors, head injury, stroke 3) Guillain-Barré syndrome 4) SSRIs, Opioids

Define Hypoglycemia

Too much insulin and too little glucose in blood causing BG level to drop <70 mg/dL (rapid onset) Causes: 1) consuming alcohol or strenuous exercise w/no food 2) not eating enough food 3) uncoordinated med/food timing 4) overmedicated (wt. loss need med adjustment) 5) using betablockers that can mask hypoglycemia symptoms 6) rapid drop of BG

Abnormal findings during an endocrine assessment include (select all that apply) a. excess facial hair on a woman b. blood pressure of 100/70 mmHg c. soft, formed stool every other day d. 3-lb weight gain over last six months e. hyperpigmented coloration in lower legs

a, e

A patient presents at the walk-in clinic complaining of diarrhea and vomiting. The patient has a documented history of adrenal insufficiency. Considering the patient's history and current symptoms, the nurse should anticipate that the patient will be instructed to do which of the following? A) Increase his intake of sodium until the GI symptoms improve. B) Increase his intake of potassium until the GI symptoms improve. C) Increase his intake of glucose until the GI symptoms improve. D) Increase his intake of calcium until the GI symptoms improve.

a

Analyze the following diagnostic findings for your patient with type 2 diabetes. Which result will need further assessment? a. A1C 9% b. BP 126/80 mmHg c. FBG 130 mg/dL d. LDL cholesterol 100mg/dL

a

Which finding indicates to the nurse that the current therapies are effective for a patient with acute adrenal insufficiency? a. Increasing serum sodium levels b. Decreasing blood glucose levels c. Decreasing serum chloride levels d. Increasing serum potassium levels

a. Clinical manifestations of Addisons disease include hyponatremia and an increase in sodium level indicates improvement. The other values indicate that treatment has not been effective.

A 27-year-old patient admitted with diabetic ketoacidosis (DKA) has a serum glucose level of 732 mg/dL and serum potassium level of 3.1 mEq/L. Which action prescribed by the health care provider should the nurse take first? a. Place the patient on a cardiac monitor. b. Administer IV potassium supplements. c. Obtain urine glucose and ketone levels. d. Start an insulin infusion at 0.1 units/kg/hr.

a. Hypokalemia can lead to potentially fatal dysrhythmias such as ventricular tachycardia and ventricular fibrillation, which would be detected with electrocardiogram (ECG) monitoring. Because potassium must be infused over at least 1 hour, the nurse should initiate cardiac monitoring before infusion of potassium. Insulin should not be administered without cardiac monitoring because insulin infusion will further decrease potassium levels. Urine glucose and ketone levels are not urgently needed to manage the patients care.

The nurse determines that additional instruction is needed for a 60-year-old patient with chronic syndrome of inappropriate antidiuretic hormone (SIADH) when the patient says which of the following? a. I need to shop for foods low in sodium and avoid adding salt to food. b. I should weigh myself daily and report any sudden weight loss or gain. c. I need to limit my fluid intake to no more than 1 quart of liquids a day. d. I will eat foods high in potassium because diuretics cause potassium loss.

a. Patients with SIADH are at risk for hyponatremia, and a sodium supplement may be prescribed. The other patient statements are correct and indicate successful teaching has occurred.

A 38-year-old patient who has type 1 diabetes plans to swim laps daily at 1:00PM. The clinic nurse will plan to teach the patient to a. check glucose level before, during, and after swimming. b. delay eating the noon meal until after the swimming class. c. increase the morning dose of neutral protamine Hagedorn (NPH) insulin. d. time the morning insulin injection so that the peak occurs while swimming.

a. The change in exercise will affect blood glucose, and the patient will need to monitor glucose carefully to determine the need for changes in diet and insulin administration. Because exercise tends to decrease blood glucose, patients are advised to eat before exercising. Increasing the morning NPH or timing the insulin to peak during exercise may lead to hypoglycemia, especially with the increased exercise.

A 55-year-old female patient with type 2 diabetes has a nursing diagnosis of imbalanced nutrition: more than body requirements. Which goal is most important for this patient? a. The patient will reach a glycosylated hemoglobin level of less than 7%. b. The patient will follow a diet and exercise plan that results in weight loss. c. The patient will choose a diet that distributes calories throughout the day. d. The patient will state the reasons for eliminating simple sugars in the diet.

a. The complications of diabetes are related to elevated blood glucose, and the most important patient outcome is the reduction of glucose to near-normal levels. The other outcomes also are appropriate but are not as high in priority.

A 54-year-old patient is admitted with diabetic ketoacidosis. Which admission order should the nurse implement first? a. Infuse 1 liter of normal saline per hour. b. Give sodium bicarbonate 50 mEq IV push. c. Administer regular insulin 10 U by IV push. d. Start a regular insulin infusion at 0.1 units/kg/hr.

a. The most urgent patient problem is the hypovolemia associated with diabetic ketoacidosis (DKA), and the priority is to infuse IV fluids. The other actions can be done after the infusion of normal saline is initiated.

The nurse is caring for a patient admitted with diabetes insipidus (DI). Which information is most important to report to the health care provider? a. The patient is confused and lethargic. b. The patient reports a recent head injury. c. The patient has a urine output of 400 mL/hr. d. The patients urine specific gravity is 1.003.

a. The patients confusion and lethargy may indicate hypernatremia and should be addressed quickly. In addition, patients with DI compensate for fluid losses by drinking copious amounts of fluids, but a patient who is lethargic will be unable to drink enough fluids and will become hypovolemic. A high urine output, low urine specific gravity, and history of a recent head injury are consistent with diabetes insipidus, but they do not require immediate nursing action to avoid life-threatening complications.

The client with a head injury has begun excreting copious amounts of dilute urine through the Foley catheter. The client's urine output for the previous shift was 3000 mL. The nurse expects that the health care provider will prescribe which medication? a. Dexamethasone b. Mannitol (Osmitrol) c. Desmopressin (DDAVP) d. Ethacrynic acid (Edecrin)

c

The nurse has administered 4 oz of orange juice to an alert patient whose blood glucose was 62 mg/dL. Fifteen minutes later, the blood glucose is 67 mg/dL. Which action should the nurse take next? a. Give the patient 4 to 6 oz more orange juice. b. Administer the PRN glucagon (Glucagon) 1 mg IM. c. Have the patient eat some peanut butter with crackers. d. Notify the health care provider about the hypoglycemia.

a. The rule of 15 indicates that administration of quickly acting carbohydrates should be done 2 to 3 times for a conscious patient whose glucose remains less than 70 mg/dL before notifying the health care provider. More complex carbohydrates and fats may be used once the glucose has stabilized. Glucagon should be used if the patients level of consciousness decreases so that oral carbohydrates can no longer be given.

A patient screened for diabetes at a clinic has a fasting plasma glucose level of 120 mg/dL (6.7mmmoL/L). Which statement by the nurse is best? a. "You will develop type 2 diabetes within 5 years." b. "You are at increased risk for developing diabetes." c. "The test is normal, and diabetes is not a problem." d. "The lab test result is positive for type 2 diabetes."

b

A patient with type one diabetes mellitus has received a prescription for beta blocker. Which issue would the nurse recognize as the major concern for this patient? a. Frequent night sweats b. lessened ability to sense hypoglycemia c. more frequent episodes of hyperglycemia d. the need for an increased basal insulin dosage

b

Patient with a diagnosis of syndrome of inappropriate antidiuretic hormone secretion (SIADH) is being cared for on the critical care unit. The priority nursing diagnosis for a patient with this condition is what? A) Risk for peripheral neurovascular dysfunction B) Excess fluid volume C) Hypothermia D) Ineffective airway clearance

b

Polydipsia and polyuria related to diabetes are primarily due to a. the release of ketones from cells during fat metabolism b. fluid shifts resulting from the osmotic effect of hyperglycemia c. damage to the kidneys from exposure to high levels of glucose d. changes in RBCs resulting from attachment of excess glucose to hemoglobin

b

The nurse is caring for a patient with Addison's disease who is scheduled for discharge. When teaching the patient about hormone replacement therapy, the nurse should address what topic? A) The possibility of precipitous weight gain B) The need for lifelong steroid replacement C) The need to match the daily steroid dose to immediate symptoms D) The importance of monitoring liver function

b

The physician has ordered a fluid deprivation test for a patient suspected of having diabetes insipidus. During the test, the nurse should prioritize what assessments? A) Temperature and oxygen saturation B) Heart rate and BP C) Breath sounds and bowel sounds D) Color, warmth, movement, and sensation of extremities

b

To monitor for complications in a patient with type 2 diabetes, which tests will the nurse in the diabetic clinic schedule at least annually (select all that apply)? a. Chest x-ray b. Blood pressure c. Serum creatinine d. Urine for microalbuminuria e. Complete blood count (CBC) f. Monofilament testing of the foot

b, c, d, f Blood pressure, serum creatinine, urine testing for microalbuminuria, and monofilament testing of the foot are recommended at least annually to screen for possible microvascular and macrovascular complications of diabetes. Chest x-ray and CBC might be ordered if the diabetic patient presents with symptoms of respiratory or infectious problems but are not routinely included in screening.

The nurse is caring for a patient at risk for an addisonian cirisis. For what associated signs and symptoms should the nurse monitor the patient? Select all that apply. A) Epitaxis B) Pallor C) Rapid respiratory rate D) Bounding pulse E) Hypotension

b, c, e

After administering Glucagon to an unconscious patient, the nurse would place the patient in which position? a. Supine b. Side-lying c. High-fowlers d. Semi-fowlers

b. nausea is a common reaction after Glucagon injection. The patient should be placed in the side lying position to prevent aspiration should the patient vomit

A patient with diabetes has an increased serum osmolality and a blood glucose level of 610 mg/dL. The nurse would monitor this patient for which complication? a. Acidosis b. Seizures c. Hyperkalemia d. cerebral edema

b. Blood glucose level more than 600 mg/dL indicates HHS, which increases the serum osmolality and produces severe neurologic manifestations, such as seizures. Acidosis is mainly associated with DKA. Fluid loss may cause mild deficits in potassium leading to hypokalemia but not hyperkalemia. Cerebral edema is complication associated with rapid administrations of IV fluids.

The nurse is caring for a 45-year-old male patient during a water deprivation test. Which finding is most important for the nurse to communicate to the health care provider? a. The patient complains of intense thirst. b. The patient has a 5-lb (2.3 kg) weight loss. c. The patients urine osmolality does not increase. d. The patient feels dizzy when sitting on the edge of the bed.

b. A drop in the weight of more than 2 kg indicates severe dehydration, and the test should be discontinued. The other assessment data are not unusual with this test.

After change-of-shift report, which patient should the nurse assess first? a. 19-year-old with type 1 diabetes who has a hemoglobin A1C of 12% b. 23-year-old with type 1 diabetes who has a blood glucose of 40 mg/dL c. 40-year-old who is pregnant and whose oral glucose tolerance test is 202 mg/dL d. 50-year-old who uses exenatide (Byetta) and is complaining of acute abdominal pain

b. Because the brain requires glucose to function, untreated hypoglycemia can cause unconsciousness, seizures, and death. The nurse will rapidly assess and treat the patient with low blood glucose. The other patients also have symptoms that require assessments and/or interventions, but they are not at immediate risk for life- threatening complications.

The nurse is preparing to teach a 43-year-old man who is newly diagnosed with type 2 diabetes about home management of the disease. Which action should the nurse take first? a. Ask the patients family to participate in the diabetes education program. b. Assess the patients perception of what it means to have diabetes mellitus. c. Demonstrate how to check glucose using capillary blood glucose monitoring. d. Discuss the need for the patient to actively participate in diabetes management.

b. Before planning teaching, the nurse should assess the patients interest in and ability to self-manage the diabetes. After assessing the patient, the other nursing actions may be appropriate, but planning needs to be individualized to each patient.

The nurse determines that demeclocycline (Declomycin) is effective for a patient with syndrome of inappropriate antidiuretic hormone (SIADH) based on finding that the patients a. weight has increased. b. urinary output is increased. c. peripheral edema is decreased. d. urine specific gravity is increased.

b. Demeclocycline blocks the action of antidiuretic hormone (ADH) on the renal tubules and increases urine output. An increase in weight or an increase in urine specific gravity indicates that the SIADH is not corrected. Peripheral edema does not occur with SIADH. A sudden weight gain without edema is a common clinical manifestation of this disorder.

A 30-year-old patient seen in the emergency department for severe headache and acute confusion is found to have a serum sodium level of 118 mEq/L. The nurse will anticipate the need for which diagnostic test? a. Urinary 17-ketosteroids b. Antidiuretic hormone level c. Growth hormone stimulation test d. Adrenocorticotropic hormone level

b. Elevated levels of antidiuretic hormone will cause water retention and decrease serum sodium levels. The other tests would not be helpful in determining possible causes of the patient's hyponatremia.

An unresponsive patient with type 2 diabetes is brought to the emergency department and diagnosed with hyperosmolar hyperglycemic syndrome (HHS). The nurse will anticipate the need to a. give a bolus of 50% dextrose. b. insert a large-bore IV catheter. c. initiate oxygen by nasal cannula. d. administer glargine (Lantus) insulin.

b. HHS is initially treated with large volumes of IV fluids to correct hypovolemia. Regular insulin is administered, not a long-acting insulin. There is no indication that the patient requires oxygen. Dextrose solutions will increase the patients blood glucose and would be contraindicated.

Which action should the nurse take after a 36-year-old patient treated with intramuscular glucagon for hypoglycemia regains consciousness? a. Assess the patient for symptoms of hyperglycemia. b. Give the patient a snack of peanut butter and crackers. c. Have the patient drink a glass of orange juice or nonfat milk. d. Administer a continuous infusion of 5% dextrose for 24 hours.

b. Rebound hypoglycemia can occur after glucagon administration, but having a meal containing complex carbohydrates plus protein and fat will help prevent hypoglycemia. Orange juice and nonfat milk will elevate blood glucose rapidly, but the cheese and crackers will stabilize blood glucose. Administration of IV glucose might be used in patients who were unable to take in nutrition orally. The patient should be assessed for symptoms of hypoglycemia after glucagon administration.

Which intervention will the nurse include in the plan of care for a 52-year-old male patient with syndrome of inappropriate antidiuretic hormone (SIADH)? a. Monitor for peripheral edema. b. Offer patient hard candies to suck on. c. Encourage fluids to 2 to 3 liters per day. d. Keep head of bed elevated to 30 degrees.

b. Sucking on hard candies decreases thirst for a patient on fluid restriction. Patients with SIADH are on fluid restrictions of 800 to 1000 mL/day. Peripheral edema is not seen with SIADH. The head of the bed is elevated no more than 10 degrees to increase left atrial filling pressure and decrease antidiuretic hormone (ADH) release.

A 38-year-old male patient is admitted to the hospital in Addisonian crisis. Which patient statement supports a nursing diagnosis of ineffective self-health management related to lack of knowledge about management of Addisons disease? a. I frequently eat at restaurants, and my food has a lot of added salt. b. I had the stomach flu earlier this week, so I couldnt take the hydrocortisone. c. I always double my dose of hydrocortisone on the days that I go for a long run. d. I take twice as much hydrocortisone in the morning dose as I do in the afternoon.

b. The need for hydrocortisone replacement is increased with stressors such as illness, and the patient needs to be taught to call the health care provider because medication and IV fluids and electrolytes may need to be given. The other patient statements indicate appropriate management of the Addisons disease.

A 56-year-old patient who is disoriented and reports a headache and muscle cramps is hospitalized with possible syndrome of inappropriate antidiuretic hormone (SIADH). The nurse would expect the initial laboratory results to include a(n) a. elevated hematocrit. b. decreased serum sodium. c. low urine specific gravity. d. increased serum chloride.

b. When water is retained, the serum sodium level will drop below normal, causing the clinical manifestations reported by the patient. The hematocrit will decrease because of the dilution caused by water retention. Urine will be more concentrated with a higher specific gravity. The serum chloride level will usually decrease along with the sodium level.

A client newly diagnosed with diabetes mellitus is instructed by the health care provider to obtain glucagon for emergency home use. The client asks a home care nurse about the purpose of the medication. What is the nurse's best response to the client's question? a. "It will boost the cells in your pancreas if you have insufficient insulin." b. "It will help promote insulin absorption when your glucose levels are high." c. "It is for the times when your blood glucose is too low from too much insulin." d. "It will help prevent lipoatrophy from the multiple insulin injections over the years."

c

A client with diabetes insipidus asks the nurse about the purpose of a new medication, vasopressin (Pitressin). The nurse explains that this medication works by which mechanism? a. Decreasing peristalsis b. Producing vasodilation c. Decreasing urinary output d. Inhibiting contraction of smooth muscle

c

A nurse caring for a patient with diabetes insipidus is reviewing laboratory results. What is an expected uninalysis finding? A) Glucose in the urine B) Albumin in the urine C) Highly dilute urine D) Leukocytes in the urine

c

A patient is prescribed corticosteriod therapy. What would be priority information for the nurse to give the patient who is prescribed long-term corticosteroid therapy? A) The patient's diet should be low protein with ample fat. B) The patient may experience short-term changes in cognition C) The patient is at an increased risk for developing infection. D) The patient is at a decreased risk for development of thrombophelbitis and thromboembolism.

c

A patient with a head injury develops SIADH. Manifestations the nurse would expect to find include: a. hypernatremia and edema b. muscle spasticity and hypertension c. low urine output and hyponatremia d. weight gain and decrease glomerular filtration rate

c

The nurse is caring for a patient with a diagnosis of Addison's disease. What sign or symptom is most closely associated with this health problem? A) Truncal obesity B) Hypertension C) Muscle weakness D) Moon face

c

The nurse is instructing a client regarding intranasal desmopressin (DDAVP). The nurse should tell the client that which occurrence is a side effect of the medication? a. Headache b. Vulval pain c. Runny nose d. Flushed skin

c

The nurse plans a class for patients who have newly diagnosed type 2 diabetes. Which goal is most appropriate? a. make all patients responsible for management of their disease b. involve the family and significant others in the care of these patients c. enable the patients to became active participants in the management of their disease d. provide the patient with as much info as soon as possible to prevent complications

c

The patient with diabetes has a serum glucose level of 824 mg/dL and is unresponsive. After assessing the patient, the nurse suspects diabetes related ketoacidosis rather than hyperosmolar hyperglycemia syndrome based on the finding of a. Polyuria b. severe dehydration c. rapid deep respirations d. decrease serum potassium

c

A college student, newly diagnosed with type one diabetes, has a headache, changes in vision, and is anxious, but does not have the portable blood glucose machine with him or her. Which action should the campus nurse advise the patient to take? a. eat a piece of pizza b. drink some diet soft drink c. eat 15g of simple carbs d. take an extra dose of rapid acting insulin

c. The fat in the pizza and the diet soft drink would not allow the blood glucose to increase to eliminate the symptoms. The extra dose of rapid acting insulin would further decrease the blood glucose.

A patient with diabetes mellitus has a blood glucose level of 680 mg/dL and minimal ketone levels in the urine. Which other finding would the nurse observe in the laboratory reports? a. Decrease blood pH b. increase potassium c. increased serum osmolality d. decrease serum bicarb

c. Blood glucose of 680 mg/dL and a minimal amount of ketones in the urine indicates that the patient has HHS; The patient will also have increased serum osmolality. Decreased blood pH and decrease serum bicarb is associated with DKA.

A nurse is caring for a 62 year old man with a history of hypertension and type 2 diabetes who has been admitted to the inpatient unit for pneumonia. The nurse enters the patients room to complete an admission assessment and notices that the patient has slurred speech and right sided weakness. After calling the rapid response team, which action would the nurse take next? a. Obtain vital signs b. obtain a crash cart c. check blood glucose d. perform a neurologic assessment

c. A patient's blood glucose levels in HHS can be very high; they increase serum osmolality and produce severe neurologic manifestations, such as aphasia and hemiparesis. It is critical to check the patients BG level for correct diagnosis and treatment because these signs and symptoms resemble those of a stroke.

An expected nursing diagnosis for a 30-year-old patient admitted to the hospital with symptoms of diabetes insipidus is a. excess fluid volume related to intake greater than output. b. impaired gas exchange related to fluid retention in lungs. c. sleep pattern disturbance related to frequent waking to void. d. risk for impaired skin integrity related to generalized edema.

c. Nocturia occurs as a result of the polyuria caused by diabetes insipidus. Edema, excess fluid volume, and fluid retention are not expected.

A 29-year-old woman with systemic lupus erythematosus has been prescribed 2 weeks of high-dose prednisone therapy. Which information about the prednisone is most important for the nurse to include? a. Weigh yourself daily to monitor for weight gain caused by increased appetite. b. A weight-bearing exercise program will help minimize the risk for osteoporosis. c. The prednisone dose should be decreased gradually rather than stopped suddenly. d. Call the health care provider if you experience mood alterations with the prednisone.

c. Acute adrenal insufficiency may occur if exogenous corticosteroids are suddenly stopped. Mood alterations and weight gain are possible adverse effects of corticosteroid use, but these are not life-threatening effects. Osteoporosis occurs when patients take corticosteroids for longer periods.

After a 22-year-old female patient with a pituitary adenoma has had a hypophysectomy, the nurse will teach about the need for a. sodium restriction to prevent fluid retention. b. insulin to maintain normal blood glucose levels. c. oral corticosteroids to replace endogenous cortisol. d. chemotherapy to prevent malignant tumor recurrence.

c. Antidiuretic hormone (ADH), cortisol, and thyroid hormone replacement will be needed for life after hypophysectomy. Without the effects of adrenocorticotropic hormone (ACTH) and cortisol, the blood glucose and serum sodium will be low unless cortisol is replaced. An adenoma is a benign tumor, and chemotherapy will not be needed.

Which statement by a nurse to a patient newly diagnosed with type 2 diabetes is correct? a. Insulin is not used to control blood glucose in patients with type 2 diabetes. b. Complications of type 2 diabetes are less serious than those of type 1 diabetes. c. Changes in diet and exercise may control blood glucose levels in type 2 diabetes. d. Type 2 diabetes is usually diagnosed when the patient is admitted with a hyperglycemic coma.

c. For some patients with type 2 diabetes, changes in lifestyle are sufficient to achieve blood glucose control. Insulin is frequently used for type 2 diabetes, complications are equally severe as for type 1 diabetes, and type 2 diabetes is usually diagnosed with routine laboratory testing or after a patient develops complications such as frequent yeast infections.

The nurse determines a need for additional instruction when the patient with newly diagnosed type 1 diabetes says which of the following? a. I can have an occasional alcoholic drink if I include it in my meal plan. b. I will need a bedtime snack because I take an evening dose of NPH insulin. c. I can choose any foods, as long as I use enough insulin to cover the calories. d. I will eat something at meal times to prevent hypoglycemia, even if I am not hungry.

c. Most patients with type 1 diabetes need to plan diet choices very carefully. Patients who are using intensified insulin therapy have considerable flexibility in diet choices but still should restrict dietary intake of items such as fat, protein, and alcohol. The other patient statements are correct and indicate good understanding of the diet instruction.

A patient who was admitted with diabetic ketoacidosis secondary to a urinary tract infection has been weaned off an insulin drip 30 minutes ago. The patient reports feeling lightheaded and sweaty. Which action should the nurse take first? a. Infuse dextrose 50% by slow IV push. b. Administer 1 mg glucagon subcutaneously. c. Obtain a glucose reading using a finger stick. d. Have the patient drink 4 ounces of orange juice.

c. The patients clinical manifestations are consistent with hypoglycemia and the initial action should be to check the patients glucose with a finger stick or order a stat blood glucose. If the glucose is low, the patient should ingest a rapid-acting carbohydrate, such as orange juice. Glucagon or dextrose 50% might be given if the patients symptoms become worse or if the patient is unconscious.

After change-of-shift report, which patient will the nurse assess first? a. 19-year-old with type 1 diabetes who was admitted with possible dawn phenomenon b. 35-year-old with type 1 diabetes whose most recent blood glucose reading was 230 mg/dL c. 60-year-old with hyperosmolar hyperglycemic syndrome who has poor skin turgor and dry oral mucosa d. 68-year-old with type 2 diabetes who has severe peripheral neuropathy and complains of burning foot pain

c. The patients diagnosis of HHS and signs of dehydration indicate that the nurse should rapidly assess for signs of shock and determine whether increased fluid infusion is needed. The other patients also need assessment and intervention but do not have life-threatening complications.

A 48-year-old male patient screened for diabetes at a clinic has a fasting plasma glucose level of 120 mg/dL (6.7 mmol/L). The nurse will plan to teach the patient about a. self-monitoring of blood glucose. b. using low doses of regular insulin. c. lifestyle changes to lower blood glucose. d. effects of oral hypoglycemic medications.

c. The patients impaired fasting glucose indicates prediabetes, and the patient should be counseled about lifestyle changes to prevent the development of type 2 diabetes. The patient with prediabetes does not require insulin or oral hypoglycemics for glucose control and does not need to self-monitor blood glucose.

The nurse is assessing a 22-year-old patient experiencing the onset of symptoms of type 1 diabetes. Which question is most appropriate for the nurse to ask? a. Are you anorexic? b. Is your urine dark colored? c. Have you lost weight lately? d. Do you crave sugary drinks?

c. Weight loss occurs because the body is no longer able to absorb glucose and starts to break down protein and fat for energy. The patient is thirsty but does not necessarily crave sugar-containing fluids. Increased appetite is a classic symptom of type 1 diabetes. With the classic symptom of polyuria, urine will be very dilute.

Endocrine

chapter 47-48

A patient with type 2 diabetes who takes oral hypoglycemics at home is admitted to the hospital with an infection ask why insulin injections have been prescribed. Which explanation would the nurse provide? a. insulin acts synergistically with the antibiotic that was prescribed b. insulin should have been prescribed for the patient to take at home c. oral hypoglycemic medications are contraindicated in patients with infection d. the infection increases the glucose level resulting in a need for more insulin

d

Patient whose lab reports shows a blood glucose level of 290mg/dL, serum bicarb of 13 mEq/L, serum potassium of 3 mEq/L, and arterial blood pH of 6 is on therapeutic management. At a follow up visit, the patient has severe hypokalemia (2 mEq/L). Which therapeutic intervention might have caused severe hypokalemia in this patient? a. Administration of 0.9% NaCl b. administration of 1 milligram of Glucagon c. administration of 5% to 10% dextrose d. administration of 0.1 U/kg/hr of insulin

d

Vasopressin (Pitressin) is prescribed for a client with diabetes insipidus. The nurse should be particularly cautious in monitoring a client receiving this medication if the client has which preexisting condition? a. Depression b. Endometriosis c. Pheochromocytoma d. Coronary artery disease

d

Which statement would be correct for a patient with type 2 diabetes who was admitted to the hospital with pneumonia? a. The patient must receive insulin therapy to prevent ketoacidosis b. the patient has islet so antibodies that have destroyed the pancreas' ability to make insulin c. the patient has minimal or absent endogenous insulin secretion and requires daily insulin injections d. the patient may have enough endogenous insulin to prevent ketosis but is at risk for hyper osmolar hyperglycemia syndrome

d

After admitting a patient with DKA to the emergency department, which nursing intervention is a priority? a. Administer IV insulin b. administer oxygen c. insert a Foley catheter d. establish IV access

d. Because fluid imbalance in a patient with DKA is potentially life threatening, the initial goal of therapy is to establish IV access and begin fluid and electrolyte replacement. Insulin is administered IV only after potassium level is determined, because insulin administration may cause hypokalemia.

The patient who is in DKA is on continuous short acting insulin infusion and is receiving continuous infusion of normal saline solution. Which assessment is the highest priority for this patient? a. Urine output b. Temperature c. respiratory rate d. cardiac monitoring

d. Continuous insulin infusion causes potassium to move from the extracellular space to the intracellular space. This shift causes a decrease in serum potassium level and can cause cardiac dysrhythmias.

Which information is most important for the nurse to communicate rapidly to the health care provider about a patient admitted with possible syndrome of inappropriate antidiuretic hormone (SIADH)? a. The patient has a recent weight gain of 9 lb. b. The patient complains of dyspnea with activity. c. The patient has a urine specific gravity of 1.025. d. The patient has a serum sodium level of 118 mEq/L.

d. A serum sodium of less than 120 mEq/L increases the risk for complications such as seizures and needs rapid correction. The other data are not unusual for a patient with SIADH and do not indicate the need for rapid action.

A 23-year-old patient is admitted with diabetes insipidus. Which action will be most appropriate for the registered nurse (RN) to delegate to an experienced licensed practical/vocational nurse (LPN/LVN)? a. Titrate the infusion of 5% dextrose in water. b. Teach the patient how to use desmopressin (DDAVP) nasal spray. c. Assess the patients hydration status every 8 hours. d. Administer subcutaneous DDAVP.

d. Administration of medications is included in LPN/LVN education and scope of practice. Assessments, patient teaching, and titrating fluid infusions are more complex skills and should be done by the RN.

Which disorder is characterized by excessive secretion of hormone vasopressin? a. Thyrotoxicosis b. diabetes insipidus c. HHS d. SIADH

d. Another term for ADH is vasopressin.

Which factor may be the cause of iatrogenic Addison's disease? a. Infarction b. Tuberculosis c. fungal infections d. adrenal hemorrhage

d. Iatrogenic Addison's disease may be caused by an adrenal hemorrhage, which is related to anticoagulant therapy, chemotherapy, ketoconazole therapy for AIDS. Infarction, tuberculosis, and fungal infections may lead to Addison's disease.

Which factor would be seen in SIADH? a. Decrease ADH b. excessive urine output c. increase serum osmolality d. increase intravascular volume

d. SIADH is characterized by an increase in ADH levels in the body which results in increased intravascular volume. Decrease ADH, excessive urine output, and increase serum osmolality are features of diabetes insipidus.

A patient with type 2 diabetes is scheduled for a follow-up visit in the clinic several months from now. Which test will the nurse schedule to evaluate the effectiveness of treatment for the patient? a. Urine dipstick for glucose b. Oral glucose tolerance test c. Fasting blood glucose level d. Glycosylated hemoglobin level

d. The glycosylated hemoglobin (A1C or HbA1C) test shows the overall control of glucose over 90 to 120 days. A fasting blood level indicates only the glucose level at one time. Urine glucose testing is not accurate reflection of blood glucose level and does not reflect the glucose over a prolonged time. Oral glucose tolerance testing is done to diagnose diabetes, but is not used for monitoring glucose control once diabetes has been diagnosed.

A 28-year-old male patient with type 1 diabetes reports how he manages his exercise and glucose control. Which behavior indicates that the nurse should implement additional teaching? a. The patient always carries hard candies when engaging in exercise. b. The patient goes for a vigorous walk when his glucose is 200 mg/dL. c. The patient has a peanut butter sandwich before going for a bicycle ride. d. The patient increases daily exercise when ketones are present in the urine.

d. When the patient is ketotic, exercise may result in an increase in blood glucose level. Type 1 diabetic patients should be taught to avoid exercise when ketosis is present. The other statements are correct.


संबंधित स्टडी सेट्स

Sociology: Ch. 20 Population, Urbanization, and the Environment

View Set

Chapter 32: The Child with Integumentary Dysfunction

View Set

Parliamentary Procedures Terms Used at a Meeting

View Set